04. PPD- Questions

Lakukan tugas rumah & ujian kamu dengan baik sekarang menggunakan Quizwiz!

The maximum area of singage on the exterior of a building is most commonly regulated by (A) building codes (B) restrictive covenants (C) state laws (D) zoning ordinances

(D) zoning ordinances

A company is considering replacing its existing heating, ventilating, and air conditioning (HVAC) system with a new system at a total cost of $55,000. It is expected that the new, more efficient system will save the company $460 per month in utility costs. The simple payback period of their investment will be ________ years. (Fill in the blank.)

10 years Payback = = ($55,000 / $460 month) / (12 months / year) = 10 years

The recommended minimum width for a sidewalk is ____ ft. (Fill in the blank.)

5 ft A 5ft width allows two wheelchairs to pass traveling in opposite directions, provides for the minimum 5ft turning diameter for wheelchairs, and generally allows other common types of use, such as two people walking side by side, people passing in opposite directions, and comfortable use of walkers and other mobility aids

A client wants a museum to be built with two identically sized stories. The museum must have 74,800 ft2 net assignable area, with a minimum of 15% of the space dedicated to circulation. The architect estimates the planned efficiency ratio to be 85%. The architect should plan for a building footprint of ________ ft2 of when doing the initial site planning. (Fill in the blank.)

44,000 ft2 The net assignable area includes space used for specific functions such as gallery space, receiving rooms, offices, work rooms, and similar spaces. It does not account for circulation, toilet rooms, mechanical rooms, space for walls, and the like. The 15% minimum circulation space is part of the 85% efficiency ratio and does not need to be included in the calculation. To determine the total gross area required, divide the net assignable area by the efficiency ratio gross area = = (net assignable area) / (efficiency ratio) = 74,800 ft2 / .85 = 88,000 ft2 The museum will have two stories of identical size, so the footprint should be half the gross area half gross = = 88,000 ft2 / 2 = 44,000 ft2

A conference room and an office are separated by a common wall that is 13ft long and 9ft high, with an STC rating of 54. The total absorption of the office has been calculated to be 220 sabins. What is the total noise reduction from the conference room to the office? (A) 46 dB (B) 57 dB (C) 64 dB (D) 72 dB

(B) 57 dB Noise reduction is calculated using the following equation: NR = TL + 10log(A/S) = 54 dB + 10log(220 sabins/ (13ft)(9ft)) = 56.74 dB

Which part of a refrigeration system heats up the most during the process? (A) Compressor (B) Condenser (C) Evaporator (D) Valve

(B) Condenser The condenser heats up most in a refrigeration system

The Owner is mostly concerned about schedule and the design criteria. Construction costs are secondary to the deadline of project completion. What Contractor compensation would an Owner be most likely to choose? (A) Cost-plus-fee with a guaranteed maximum price (B) Cost-plus-fee (C) Stipulated sum (D) Unit price

(B) Cost-plus-fee Typically an Owner may lean more towards cost-plus-fee with a GMP but in this case, cost is not a concern. It's less likely that the Owner will run into conflicts if they remove the ceiling on how much the Contractor can make

A zoning map is most likely to be separated into _______ (A) Topographic areas (B) Districts (C) Construction type (D) Economic vitality

(B) Districts The majority of zoning codes are separated into districts. Some common districts include: residential, commercial, industrial, and institutional. These are often abbreviated and broken down into smaller categories (Residential may include R1, R2, and R3 districts)

Radon is a developed from what source? (A) Natural radioactivity in soils (B) Pockets of gas in the ground (C) Off-gassing materials located in the soil (D) Contaminated groundwater

(A) Natural radioactivity in soils Radon is a developed from natural radioactivity in soils

What kind of glazing should the architect specify on an interior door french door to be most economical and comply with the building code? (A) Tempered (B) Insulated (C) Laminated (D) Low-E (E) Single-pane

(A) Tempered Tempered glass is the correct answer. Glass in doors is required to be safety glass which includes tempered and laminated. Tempered is typically less expensive because it includes only one pane of glass where laminated includes two

Which statement is true about dry standpipes? (A) They're used by firefighters who have to pump water into them (B) They're used to drain floors of excess water during fire fighting (C) They are filled with Halon gas or carbon dioxide and triggered automatically (D) They are used by building occupants to put out a fire before it gets out of control

(A) They're used by firefighters who have to pump water into them

An architect in Richmond, Virginia, has been asked to design a replacement roof for a hospital. The existing roof has a slope of approximately 1:12. the building supervisor requests a system that allows for additional insulation to be installed and includes paths of pavers for easy access to mechanical units and other equipment located on the roof. Which type of single-ply roofing system should be recommended? (A) fully adhered ethylene propylene diene monomer (EPDM) (B) loose-laid EPDM (C) fully adhered polyvinyl chloride (PVC) (D) mechanically attached PVC

(A) fully adhered ethylene propylene diene monomer (EPDM)

If wood has a moisture content of 30%, it is generally referred to as being _________ (A) green (B) wicked (C) ripe (D) dry

(A) green If wood has a moisture content of 30% or greater it is referred to as being green. This means that is has not had a chance to season or dry yet. Wood with a moisture content less than 20% will not rot if it maintained below that percentage

A tactile finish should be applied to hardware on a door that leads to a building's (A) fire stairs (B) boiler room (C) restrooms (D) exterior

(B) boiler rooms A tactile finish (rough surface) is applied to hardware on doors leading to building areas that would be dangerous for a person with impaired vision, such as a boiler room

Which of these are important considerations in designing a fire-rated ceiling? (Choose the two that apply.) (A) hold-down clips (B) structural slab (C) thermal insulation (D) composition of the floor/ceiling assembly (E) sound absorption (F) style of grid

(A), (D)

Which paint type would serve best as an anti-graffiti coating? (A) acrylic (B) alkyd (C) oil (D) urethane

(D) urethane Urethane is a high-performance coating and has superior resistance to abrasion, grease, alcohol, water, and fuels. It resists the adhesion of graffiti to surfaces and allows for relatively easy removal of graffiti

What type of electronic security system best protects the exterior of an office building? (A) audio alarms (B) motion sensors (C) thermal detectors (D) video surveillance

(D) video surveillance

Next to one of the items in the Contractor's bid it says (Allowance). What is an allowance? (A) A maximum price not to be exceeded for that item (B) A minimum price not to be exceeded for that item (C) A way for Contractors to allocate funds to items that have may not have been clearly specified (D) A way for Contractors to identify items beyond their scope

(C) A way for Contractors to allocate funds to items that have may not have been clearly specified The establishment of cash allowances in construction contracts is a convenient method of allocating construction funds to portions of the work that cannot be specified with sufficient particularity for competitive bidding at the time of contracting

Interior building mold is a result of what kind of conditions? (A) Hot (B) Dry (C) Damp (D) Cold

(C) Damp When mold spores drop on places where there is excessive moisture, such as where leakage may have occurred in roofs, pipes, walls, plant pots, or where there has been flooding, they will grow. Many building materials provide suitable nutrients that encourage mold growth. Wet cellulose materials, including paper and paper products, cardboard, ceiling tiles, wood, and wood products, are particularly conducive for the growth of some molds. Other materials such as dust, paints, wallpaper, insulation materials, drywall, carpet, fabric, and upholstery, commonly support mold growth

To provide proper drainage for streets in an economical manner, what should be placed at the center of the streets? (A) Swale (B) Gutters (C) Pervious surfaces (D) Crown

(D) Crown Crowns are high points placed at the center of streets to provide proper drainage to a gutter or swale at the street edge

Carpet durability is most directly correlated to which of the following? (A) Loop and pull (B) Color (C) Pile height (D) Pile density

(D) Pile density Pile density refers to the quantity of yarn per unit of carpet. More yarn equates to a more durable carpet

Which of the following is a method of rating wall sections or doors according to their typical or overall resistance to sound transmission? (A) NR (B) PWL (C) SPL (D) STC

(D) STC STC or Sound Transmission Coefficient is a single number rating scale that measures a wall, ceiling, or floor assembly's ability to block sound transmission

What is the unit for sound absorption? (A) Decibel (B) Perm (C) Refraction (D) Sabin

(D) Sabin The unit for sound absorption is the sabin

What is the term for an opening between wood growth rings containing resin? (A) Peck (B) Pitch pocket (C) Wane (D) Warpage

(B) Pitch pocket An opening between wood growth rings containing resin is termed a pitch pocket

What ingredients are in glass? (Choose all that apply.) (A) Portland cement (B) Lime (C) Sand (D) Soda ash (E) Water

(B), (C), (D) Of the listed items lime, sand, and soda are ingredients in glass

An old warehouse is being converted into an apartment building. The project will be seeking LEED certification, so a high priority will be placed on sustainable design. Which of the following existing building elements is most likely to be reused? (A) masonry walls (B) asbestos insulation (C) roofing membranes and asphalt shingles (D) windows

(A)

What is the approximate current in a 120V circuit serving nine 150W downlights? (A) 11 A (B) 15 A (C) 20 A (D) 26 A

(A) 11A The power in the circuit is 9x150W = 1350W. The equation for power in AC circuits is P=VI(pf) Where (pf) is a power factor. Because the circuit only has resistive loads (incandescent lights), use a power factor of 1.0. Hence, I = P/V(pf) = 1350W/(120V)(1.0) = 11.25 A

What is the occupant load for a restaurant dining room that is 2,500 ft2 in area? (A) 167 occupants (B) 184 occupants (C) 202 occupants (D) 222 occupants

(A) 167 occupants Use table IBC 1004.1.2 Occupant load = = (area/IBC occupant load factor) = (2,500 ft2) / (15 ft2/occupant) = 167 occupants

An office building is being built with Type II-B construction and a sprinkler system. Each floor will have the same area. The allowable area factor increase has been determined to be 25%. What is the maximum allowable area for the building? (A) 300,000 ft2 (B) 370,000 ft2 (C) 460,000 ft2 (D) 550,000 ft2

(A) 300,000 ft2 Aa == maximum allowable area At == allowable area factor NS == allowable area factor (non-sprinkler) If == area factor increase Sa == number of stories Multiple story calculation: Aa = = (At + NS*If) (Sa) = (69,000 + 23,000*.25) * 4 = 299,000 ft2

What is the equivalent thickness of a concrete block with a nominal thickness of 8" that is 60% solids? (A) 4.58" (B) 5.50" (C) 6.60" (D) 7.91"

(A) 4.58" The actual thickness of the block is 7-5/8". The equivalent thickness is (7.625")(0.60) = 4.58"

Which of the following would have the greatest impact on the size and configuration of an accessible restroom? (A) 5 ft clear circular turnaround space (B) maneuvering space on the outside of the entry door to the room (C) clear space at towel dispensers and full-height mirrors (D) minimum 36 in access route into and through the room

(A) 5 ft clear circular turnaround space Providing for a 5 ft turning circle requires the most space of the four choices listed. If the turning circle is provided, it is very likely that a 36 in access space and clear space at the towel dispensers will also be available. The maneuvering space on the outside of the entry door is irrelevant to the interior dimensions of the restroom

A high school teacher is standing at the front of a classroom delivering a lecture to a class of 25 students. Students in the front row, 3 feet in front of the teacher's lectern, perceive the teacher's voice at 65 dB. What is the decibel level of the teacher's voice in the back row, 12 feet from the lectern? (A) 53 dB (B) 56 dB (C) 59 dB (D) 65 dB

(A) 53 dB The sound intensity level at the back row of the classroom is 53 dB. Sound intensity levels decrease 6 dB for every doubling of distance. 3' doubled is 6', and 6' doubled is 12' Hence the total drop in decibels is 12. 65 dB - 12 dB = 53 dB

One office machine produces 70 dB of sound, and a second machine generates 76 dB. What is the combined sound intensity level of the two office machines? (A) 77 dB (B) 95dB (C) 118 dB (D) 146 dB

(A) 77 dB Use the table that shows which value to add to the overall noise based on the difference between the two sources. Since the difference is 76-70=6 dB, the table shows that 1 dB is to be added to the higher value. Hence, 77 dB

Which parking configuration is the most difficult for a driver to maneuver within? (A) 90 degree (B) 60 degree (C) 45 degree (D) 30 degree

(A) 90 degree The most difficult parking configuration for a driver to maneuver within is a 90 degree angle arrangement. 90 degree parking permits two-way aisles 45 and 60 degree parking only permits one-way aisles, but are relatively economical and allow for easy access 30 degree parking is the least efficient and seldom used

What is the lightest metal roofing material? (A) Aluminum (B) Copper (C) Iron (D) Steel

(A) Aluminum Per cubic foot, aluminum is the lightest metal widely used in building construction In pounds per cubic foot: + Aluminum = 168 + Iron = 490 + Steel = 490 + Copper = 560

An architect is asked to design an office suit that limits sound transmission into the surrounding corridor. Which of the following is the most important acoustical strategy to include in the design? (A) Build walls with staggered studs and mount the gypsum board on resilient channels (B) Extend the partitions from deck to deck and provide acoustical seals at the top and bottom of the walls (C) Provide unfaced batt insulation in the partitions between the office and the corridor (D) Specify absorptive finish materials such as carpeting, draperies, and acoustical tile

(A) Build walls with staggered studs and mount the gypsum board on resilient channels

A dynamic load is one that _______ (A) Changes rapidly (B) Changes slowly (C) Does not change over time (D) Is applies slowly

(A) Changes rapidly A dynamic load is one that changes rapidly such as wind, users, or an elevator

Which term refers to a steel beam and a concrete slab that are connected so that they act together as a single structural unit to resist bending stresses? (A) Composite beam (B) Composite load (C) Consteel beam (D) Dual-type beam

(A) Composite beam When two or more materials are working together as a beam they are considered a composite beam

What is the heat transfer process that occurs when objects are in direct contact? (A) Conduction (B) Convection (C) Induction (D) Radiation

(A) Conduction Conduction is the process by which heat or electricity is directly transmitted through a substance when there is a difference of temperature or of electrical potential between adjoining regions, without movement of the material

An architect is laying out the house sewer for a small commercial building. The sewer has been sized at a 6" diameter with a 1/8"/ft slope. The architect discovers that the original information on the invert of the main sewer line, which is approximately 300' away, was incorrect and that it is actually 1 ft higher than planned. In order to minimize cost, what is the best course of action? (A) Decrease the slope and increase the size of the house sewer (B) Angle the house sewer to connect farther down the main line (C) Change the location of the building (D) Decrease the slope of the sewer and add an intermediate manhole

(A) Decrease the slope and increase the size of the house sewer

Many problems associated with exterior insulation and finish systems (EIFS) can be solved using which of the following design techniques? (A) Design the wall using the rain screen principle (B) Use expansion joints at a maximum spacing of 10 ft (C) Increase the thickness of the finish coat (D) Provide extra flashing at window and door joints

(A) Design the wall using the rain screen principle A standard EIFS is designed as a barrier against moisture. The level of moisture prevention depends on the finish and the proper construction of joints and details. An EIFS can experience problems if water leaks behind the finish and insulation and becomes trapped, damaging framing and other building components. Some proprietary systems are available that incorporate the rain screen principle by using a mesh or some other means of allowing pressure to equalize outside and inside of the system. Any water that does leak through is drained to the outside through weep holes The other common problem with a standard polymer-based (PB) EIFS is puncturing or denting. This can be addressed by using a polymer-modified system (PM) or by using a high-impact PB system with fiberglass mesh and an extra layer of base coat

Which of the following include the area of exterior walls? (A) Gross square footage (B) Gross assignable square footage (C) Net square footage (D) Net assignable square footage

(A) Gross square footage Gross assignable square footage is sometimes incorrectly used, but it is not an actual way to calculate square footage. Inherently the term gross includes everything so it's a bit of an oxymoron to use gross assignable square footage

Which type of lumber is most often used for structural purposes? (A) Hardwood (B) Hardwood and softwood are used equally often (C) Hardwood and softwood use changes based on the season (D) Softwood

(A) Hardwood Softwood, wood from mainly coniferous trees, is most often used for structural purposes. Softwood and hardwood don't necessarily describe the strength or softness of the wood

When designing a research laboratory building, how might an architect provide for extensive mechanical system requirements as well as a programmed requirement for flexibility in changing the layout of the laboratories over time? (A) Use interstitial spaces (B) Locate mechanical risers on the outside of the building (C) Distribute mechanical rooms throughout the building (D) Design a compact, multistory structure

(A) Use interstitial spaces Mechanical risers on the outside of the building would not provide for changes to the distribution network within the building Distributing mechanical rooms might allow for one to be upgraded as necessary without disturbing other parts of the building, but would not provide for easy modification of mechanical and electrical services to individual rooms Although a compact building would reduce the length of service runs for mechanical and electrical systems, it would still require disruption of occupied spaces while changes were being made Interstitial space is intermediate space between floors of a building used for mechanical, plumbing, and electrical systems. Interstitial space allows for the installation and modification of mechanical systems independent from the usable floors in between. It is often used for laboratory buildings and hospitals where the requirements for mechanical and electrical systems are extensive and often require changes and additions as the needs of the building and technology change. Although it increases initial cost, the inclusion of interstitial spaces provides the most flexibility for accommodating changes

Which of the following domestic water heating systems would yield the lowest operating costs for a duplex residential unit? (A) active closed-loop solar (B) direct-fired storage tank (C) ground-source heat pump (D) tankless instantaneous

(A) active closed-loop solar A solar water heating system would have the lowest long-term operating cost because no fuel costs would be involved A standard direct-fired storage tank unit would require purchasing gas, oil, or some other type of fuel to heat the water. A ground-source heat pump would not be able to supply water hot enough for domestic use and is inappropriate for this application; ground-source heat pumps are used for heating the home, not hot water A tankless instantaneous unit would incur ongoing expenses due to its use of electricity and is not appropriate for general household domestic hot water generation

For short spans and in relatively small openings 8" and 12" thick brick walls, the most commonly used steel lintels are (A) angles placed back to back (B) channels or C-shapes (C) W-shapes used for beams (D) structural T-shapes

(A) angles placed back-to-back In short spans and relatively small openings in 8" and 12" thick brick walls, the most commonly used steel lintels are angles placed back-to-back. Three steel angles are sometimes used. The W-shape beam is often used for longer spans or thicker walls

What is a requirement for a door opening in a masonry partition? (A) bond beam (B) arch action (C) weep holes (D) flashing

(A) bond beam A bond beam is a masonry unit made to accommodate reinforcing and grout to span openings in masonry walls. These are often used in place of steel lintels

Which mortar joint is recommended for exterior use? (A) concave (B) trowel struck (C) weather struck (D) raked

(A) concave Trowel struck joints are not recommended because water can accumulate on top of the brick and seep into the joint A weather struck joint is sometimes used, but water running down the brick above the joint may not drop off and may instead run horizontally under the brick Raked joints are not recommended as water can pool in the tiny void between bricks, seep into the pores of the materials, and eventually damage the masonry and mortar A concave joint avoids all these problems and has the added advantage of being easily tooled, which compresses the mortar

High voltages are used in commercial buildings because (A) conductors and conduit can be smaller (B) a wider variety of loads can be accommodated (C) commercial buildings require more power (D) transformers can step down the voltages to whatever is required

(A) conductors and conduit can be smaller As voltages increase, current may be decreased and the system will still provide the same amount of power. Lower currents require smaller conductors. For large commercial buildings, smaller conductors translate to less expense in conductors and conduit, as well as easier installation of smaller wires

What do transformers do? (Choose the two that apply.) (A) Decrease alternating current voltages (B) Change AC to DC (C) Change DC to AC (D) Increase alternating current voltages

(A), (D) A transformer increases and decreases current voltages between two or more circuits

A soils report indicates that bentonite is present below the site of a proposed two-story manufacturing building. What type of foundation system should be used? (A) drilled piers with grade beams (B) raft foundation (C) caissons with pile caps (D) extended spread footings

(A) drilled piers with grade beams Bentonite is an expansive type of clay that can push foundations and floor slabs upward when it gets wet. To prevent this, drilled piers are used to support the building weight on bedrock or stable soil below the bentonite. Grade beams span continuously between the piers and transmit the building loads from the superstructure to the piers. Voids are left below the grade beams to allow the bentonite to expand without transmitting uplift forces A raft foundation is used to distribute a building load over a large area of low-bearing capacity coil Caissons with pile caps are used to distribute a load from one column to two or more caissons or piers and would only be appropriate if there was a void below the pile cap Spread footings placed on bentonite would be subject to the uplift of the swelling clay soil and would not be appropriate

The maximum allowable area of a building is limited by a combination of (A) floor area ratio and construction type (B) occupancy group and setback requirements (C) bulk plane limits and floor area ratio (D) construction type and setback requirements

(A) floor area ratio and construction type Zoning regulations limit total building area based on a floor area ratio and setbacks, while building codes limit building area by construction type and occupancy group Bulk plane limits may affect the area by limiting height in some cases, but they are not a primary determinant Of the options given, the floor area ratio (from zoning codes) and construction type (from building codes) is the combination that limits maximum area

Which of the following building types has the highest percentage of total construction cost allocated for HVAC? (A) full-service, sit down restaurant (B) climate controlled mini-warehouse (C) hospital (D) parking garage

(A) full-service, sit down restaurant

In what type of building is a 120/208 voltage, three phase power appropriate? (A) industrial (B) small commercial (C) residential (D) large commercial

(A) industrial

What types of buildings are most suited for a pan joist concrete system? (A) industrial and storage buildings (B) residential buildings (C) hotels (D) office buildings

(A) industrial and storage buildings

Which of the following types of glass is the strongest? (A) laminated (B) annealed (C) tempered (D) heat-strengthened

(A) laminated Laminated glass consists of two or more layers of glass with a layer of plastic bonded in between. If the glazing is broken, the pieces of glass will be held together by the layer of plastic. Laminated glass is used in applications such as bulletproof glazing, car windshields, and skylights Annealed glass, or ordinary window glass, is made by floating molten glass on top of molten tin. As the liquid moves through the production process, it is slowly cooled into a perfectly flat sheet of solid glass. Annealed glass may be subjected to processes such as tempering and heat strengthening to change its characteristics Tempered and heat-strengthened glass are both formed by heating annealed glass to very high temperatures. Heat strengthened glass is heated and then cooled slowly. Tempered glass is heated to higher temperatures and then cooled quickly. Tempered glass is about twice as strong as heat-strengthened glass and about four times as strong as annealed glass. Tempered glass is used in glass doors and windows, as shelving, and for many other uses where a safety glass is required

Which of the following determines minimum parking requirements for a specific site? (A) local zoning ordinances (B) building codes (C) owner's preferences (D) ADA/ABA Guidelines

(A) local zoning ordinances

Which type of glass is the most appropriate choice for a location in the northeastern United States? (A) low-e (B) reflective (C) tinted (D) triple-pane

(A) low-e

Which of the following is most likely to be found in a local zoning ordinance? (A) minimum setbacks from the property lines (B) maximum occupancy (C) fire-rated assemblies (D) types of materials that may be used for construction

(A) minimum setbacks from the property lines The others are all found in the IBC

An architect can best discourage criminal activity in a street-level apartment lobby by applying which of the following design strategies? (A) separating the lobby and the street with a large expanse of glass (B) changing the paving texture at the property line (C) clearly marking the entrance with a "Residents Only" sign (D) adding bright light over the entrance door

(A) separating the lobby and the street with a large expanse of glass Although all these design features would help discourage criminal activity, the best approach would be to make activity inside the lobby visible from the street. This employs the principle of natural surveillance

The most important characteristic of an on-site road designed to serve parking areas and service vehicles is the (A) slope (B) tangent (C) horizontal alignment (D) vertical alignment

(A) slope

Which natural cooling technique is utilized by the Pantheon? (A) stack ventilation (B) pools of water (C) thermal mass (D) cross ventilation

(A) stack ventilation The Pantheon has an oculus (open air hole) in the center of its dome The Pantheon is also a large thermal mass, however, so this question is a bit bogus

Corners and eaves are generally (A) subject to higher wind forces (B) subject to lower wind forces (C) subject to the same amount of wind forces as the rest of the building (D) not affected by wind forces

(A) subject to higher wind forces Corners and eaves generally receive higher wind forces than the rest of a building. This is why most wind damage occurs at corners and eaves

Problems with veiling reflections in a general-purpose workroom could best be reduced by (A) substituting ambient light for direct light fixtures (B) repositioning the light fixtures (C) reducing the brightness of the light fixtures (D) changing the type of lamps

(A) substituting ambient light for direct light fixtures A veiling reflection is glare caused when the image of a light source is reflected from a viewing surface into the eye. The best way to reduce it is to provide general background illumination (ambient lighting) so the light sources are not concentrated in the area of the lamp Repositioning the luminaires (or the task) can reduce veiling reflections, but only when the task is in a specific location in relation to the light source. In a general-purpose workroom this would not be feasible. Reducing the brightness of the light source would help, but would also reduce the illumination provided for the task. Changing the type of lamps would have little effect on reflected glare

Reinforced concrete or masonry retaining walls are usually necessary when (A) the height of the wall exceeds 4' (B) expansive clay soil is present (C) the groundwater level is above the lowest exposed portion of the wall (D) drainage behind the retaining wall is a problem

(A) the height of the wall exceeds 4' Retaining walls less than 4' high can usually be constructed of any suitable material. Higher walls become subject to sliding and overturning forces and must be engineered to resist the expected loads. Typically these engineered retaining walls are constructed of concrete or masonry and are built on footings The type of soil, the groundwater level, and the soil porosity alone do not dictate the use of reinforced concrete or masonry

Several sites are being considered for the construction of a new community college. The most appropriate site would be the one adjacent to (A) two major intersecting highways (B) libraries and shopping (C) a technology-oriented office campus (D) a residential neighborhood and public transportation

(A) two major intersecting highways A community college is a regional resource and would benefit from easy access to transportation sources encompassing a wide area, which two highways would provide Although public transportation is also a requirement, a large-scale development like a community college would not be appropriate for a residential neighborhood This problem requires an understanding of whether the proposed land use is local, district, or regional in nature and what the compatible uses are. A local based project, such as a branch library or church is compatible with residential uses and pedestrian transportation, while regionally based projects require public or vehicular transportation

Galvanic action can be avoided by (A) using neoprene spacers (B) increasing the thickness of the materials (C) reducing contact with dripping water (D) all of the above

(A) using neoprene spacers Dissimilar metals should be physically separated by nonconducting materials such as neoprene in order to prevent galvanic action Increasing the thickness of the materials may postpone their complete deterioration but will not prevent it, so option B is incorrect. Direct contact with water will speed up galvanic action, but even moisture in the air is sufficient to cause it, so option C is incorrect

Galvanic action can be avoided by (A) using neoprene spacers (B) increasing the thickness of the materials (C) reducing contact with dripping water (D) using a third metal spacer

(A) using neoprene spacers Dissimilar metals should be physically separated by nonconducting materials such as neoprene in order to prevent galvanic action The other three options would not entirely avoid galvanic action

Which of the following engineered products would be best to use in place of traditional wood joists for spans from 16' to 20'? (A) wood I-joists (B) glued-laminated members (C) medium-density fiberboard (D) parallel-chord wood trusses

(A) wood I-joists Wood I-joists are designed to replace standard solid wood joists and rafters and would be very efficient, in terms of both cost and structure, for the spans indicated. Glulam members would be more expensive and heavier than necessary for standard floor or roof framing in these span ranges Medium-density fiberboard is a panel product and is not designed for structural uses such as beams or joists Parallel-chord wood trusses could be used, but they are more efficiently used for longer spans

Which elements are volatile organic chemicals primarily composed of? (Choose the two that apply.) (A) Carbon (B) Hydrogen (C) Oxygen (D) Chlorine (E) Helium

(A), (B) Volatile organic chemicals (VOCs) are primarily composed of carbon and hydrogen. Methane (CH4) is the most abundant VOC in the atmosphere

What is the minimum slope for on-grade drainage? (A) 1% (B) 2% (C) 4% (D) 8%

(B) 2% 2% is the minimum slope for on-grade drainage. Any slope less than this may not drain water away from the site properly

In a hot-arid climate in the northern hemisphere, which design techniques would be most appropriate? (Choose the three that apply.) (A) Deep overhangs (B) Thick walls (C) High ceilings (D) Skylights (E) Planting deciduous trees on the north side of the building

(A), (B), (C) Deep overhangs have the ability to shade more of the sun during warmer summer months Thick walls slow the heat radiation to the interior from the sun beating on exterior walls. High ceilings give room for hot air to rise, making the lower air cooler. Skylights would allow the sun's radiant heat to enter directly into the space making it hotter. Deciduous trees on the north side of the building would have little effect on building temperature. To be effective passive cooling solutions deciduous trees should be placed on the south and southwest sides of a building

What items are reflected on a topographic survey? (Choose all that apply.) (A) Existing topography (B) Existing site features (C) Site datum point (D) Dimensions of proposed building slab (E) Location of smoke detectors (F) Windows (G) Interior walls

(A), (B), (C) The purpose of a topographic survey is to document the existing site conditions Of the listed items, this would include: existing topography, existing site features, and site datum point A topographic survey is done before any proposed changes are made, so a proposed slab wouldn't show up. Features specific to the building such as smoke detectors, windows, and interior walls would not show up either

Where should weep holes be located in a brick wall? (Choose the three that apply.) (A) at the lowest course of brick (B) above windows (C) above shelf angles (D) below window sills (E) under copings (F) every 15' vertically

(A), (B), (C) Weep holes should be located where water may accumulate within a multi-wythe, cavity, or veneer wall. The weep holes allow the water to drain or be wicked out of the wall cavity Water tends to accumulate at the bottom of a wall or where any penetration through the wall creates a "shelf," such as above a window or at a steel angle Both flashing and weep holes at 24" on center minimum, should be provided at each of the locations listed

What type of glass would be appropriate for a 10-story building? (Choose the four that apply.) (A) tempered (B) heat-strenthened (C) laminated (D) annealed (E) fire-rated (F) float

(A), (B), (C), (E) Annealed glass is the standard glass used in most noncritical glazing situations Float glass is the same as annealed glass and would not be strong enough to be used All the other types of glass listed have greater strengths and could be used in a tall building with large panels of glass subject to high wind loads and thermal cycling

Select which items are usually present in the geotechnical engineer's subsurface report. (Choose all that apply.) (A) Results of field tests (B) Results of laboratory tests (C) Recommended types of foundations (D) Soil sieve analysis (E) Saturation point (water table) (F) Recommended treatment for contaminated soil

(A), (B), (C), (E) The geotechnical engineer will conduct field and laboratory tests to determine the soils and their characteristics. They provide this information in their report. Based on their findings, they can usually determine the level of the water table and they will be able to recommend a foundation type. The structural engineer may or may not use this recommendation. A soil sieve analysis is not typically a part of what a soils engineer provides. If contaminated soils are found, conditions need to be assessed by a soil remediation expert.

Select all of the regulatory documents that may affect a building's design (A) Building codes (B) Zoning ordinances (C) Historic districts (D) Use codes (E) FAA (F) Watersheds (G) US parking code

(A), (B), (C), (E), (F) The "use code" and "US parking code" are fictional Building codes and zoning ordinances are a standard part of any architectural project. Depending on the location, regulations for historic districts, watersheds, and the FAA (Federal Aviation Administration) may come into play as well

Which of the following are tenants of Crime Prevention Through Environmental Design (CPTED)? (Choose the four that apply.) (A) Provide clear transitions between public and private spaces (B) Provide windows for tenant surveillance (C) Make a clear distinction between public and semi-public spaces (D) Place activity in a safe location (E) Encourage the use of keypad locks (F) Locate stairways near heavily used areas

(A), (B), (C), (F)

Which site design elements for fire protection are the responsibility of the architect? (Choose the four that apply.) (A) position and size of building canopies (B) surface material outside of the building (C) traffic control fences and bollard positions (D) maximum fire hydrant spacing on the street (E) Utility poles and overhead utility lines (F) width of fire apparatus access drives

(A), (B), (C), (F)

Which of the following strategies would be employed in a community adopting the principles of defensible space? (Choose the four that apply.) (A) gated streets (B) strict enforcement of code regulations (C) loan programs for first-time homebuyers (D) concentration of all housing in one area (E) specification of vandal-resistant materials (F) clear distinctions between public and private spaces

(A), (B), (C), (F) Defensible space is a theory that is based on the principles developed by Oscar Newman Gated streets along are insufficient; defensible space requires stricter enforcement of building codes (including criminal charges for noncompliance), help for new homeowners to finance and maintain their properties, and community programs designed to facilitate interaction among neighbors

An architect is planning a 100,000 FT2 university classroom building. The mechanical engineer estimates that the total floor area required for the boiler room and chilled water plant will be about 3,000 FT2. Which of the following criteria should also be kept in mind when determining the location and design of the mechanical rooms? (Choose the three that apply.) (A) Each mechanical rooms should have at least one exterior wall (B) The boiler room should be adjacent to the chilled water plant (C) Rooms should be as square as possible (D) Ceilings in both rooms should be at least 12' high (E) Mechanical rooms must be placed on the ground floor (F) The mechanical rooms should be equal in size

(A), (B), (D)

Part of the classifications for wood are based on the manufacturing processes that were used to create the lumber. What are the common classifications? (Choose the three that apply.) (A) Dressed (B) Finished (C) Rough (D) Smooth (E) Stained (F) Worked

(A), (C), (F) + Dressed lumber is surfaced by a planing machine to attain a smooth surface and uniform size + Rough lumber is not dressed surfaced but sawed, edged, and trimmed to the extent that saw marks show in the wood on the four longitudinal surfaces of each piece for its overall length + Worked lumber is dressed and also matched, shiplapped, or patterned.

What are characteristics of a 'wythe'? (Choose the three that apply.) (A) Continuous wall section (B) May be one masonry unit in thickness (C) Segmented (D) May be two masonry units in thickness (E) More vertical than horizontal

(A), (B), (D) A wythe is a continuous vertical section of masonry one unit in thickness. A wythe may be independent of, or interlocked with, the adjoining wythe(s). A single wythe of brick that is not structural in nature is referred to as a veneer A multiple-wythe masonry wall may be composed of a single type of masonry unit layered to increase its thickness and structural strength, or different masonry units chosen by function, such as an economical concrete block serving a structural purpose and a more expensive brick chosen for its appearance While a wythe is considered a continuous vertical, there is no requirement for one to be taller than it is wide to maintain its classification. Segments are not a characteristic of wythes

Which of the following would help to minimize oil canning? (Choose the three that apply.) (A) Design attachment hardware that allows panels to move in response to expansion and contraction caused by changes in the temperature (B) Include information in the specifications that requires the installer to transport panels vertically rather than horizontally (C) Specify a high-gloss painted finish on the panels (D) Carefully coordinate the design of the supporting structure to ensure that it is level and plumb (E) Design the panels to allow for a slight bend in the metal (F) Specify a thicker panel than would be normally required

(A), (B), (D) Oil canning gives a metal siding panel a wavy appearance. Generally, it is not a structural issue, just an aesthetic one. However, it can be minimize through careful design of the panels, attachment hardware, and supporting structure. For example, a textured, ribbed, or matte finish will minimize the appearance of waviness more than a smooth, glossy finish. Allowing space at the hardware connections for expansion and contraction will also help to minimize the waviness Most of the things that can be done to minimize oil canning fall under the responsibility of the contractor in the field, but the techniques can be written into the architect's specifications to ensure proper handling of the materials and installation. Panels should always be transported vertically rather than horizontally, and care should be taken not to twist them. The supporting structure should be as flat, or planar, as possible

Which of the following statements about steel connections are true? (Choose the four that apply.) (A) Bolts are easily installed, inexpensive, and can be visually checked (B) One of the advantages of using welded studs is a reduction in the number of holes to be punched (C) The overall cost of riveted construction is usually lower than the cost of using bolted connections (D) Welding is more practical for moment connections (E) Rivets are still fabricated on a limited basis (F) Welding is less efficient than bolted construction

(A), (B), (D), (E) Bolts are relatively inexpensive and easy to install. Welded studs, in which one end of a bolt is welded to one piece of steel, minimize the number of holes that must be punched for bolts. Moment connections require a continuous connection between two pieces of steel, and welding achieves that continuous connection better than the use of bolts Rivets, though once popular, are not often used today, but they are still fabricated on a limited basis. The overall cost of riveted construction is usually higher than the cost of construction using bolted or welded because of increased labor and equipment requirements. Welding is often more efficient than bolted construction because there are no angles, bolts, or washers to deal with and no clearance problems with wrenches

Which of the following generally is required to comply with a site setback? (Choose the four that apply.) (A) bay window (B) roof overhang (C) fence (D) detached garage (E) landscaping (F) deck

(A), (B), (D), (F) Most zoning setback regulations affect buildings, accessory buildings, and their various components. These include bay windows, roof overhangs, and decks Each jurisdiction interprets setback requirements a little differently, so it is important to research the requirements governing a specific site

Certified contractors are required for handling or abatement of which of the following hazardous materials? (Choose the four that apply.) (A) asbestos (B) lead (C) polyvinyl chloride (PVC) (D) polychlorinated biphenyls (PCB) (E) radon (F) vermiculite

(A), (B), (D), (F) Radon detection and remediation can be done by anyone, from homeowners to specialty contractors PVC can be removed by anyone Asbestos, lead, PCBs, and vermiculite all must be handled by a certified contractor

Which of the following factors affect the wind pressure on a building? (choose the four that apply.) (A) wind speed (B) terrain surrounding the building (C) soil type (D) building height (E) building wieght (F) building shape

(A), (B), (D), (F) Wind pressure on a building is affected by wind speed, surrounding terrain, and the building's height and shape The weight and soil type do not affect it

What type of effects can excess moisture have most on indoor environmental quality? (Choose the three that apply.) (A) Growth of bacteria (B) Growth of fungi (C) Shortness of breath (D) Burning eyes (E) Growth of dust mites

(A), (B), (E) Excess moisture can increase the growth of fungi, bacteria, and dust mites. Excess moisture is often created from cooking, washing, and exhaling. It can be mitigated or resolved through exhaust ventilation and dehumidification

A preliminary drainage analysis for a small land parcel in a cold, wet location indicates slopes of 1% or less. The land parcel is intended to be developed into a suburban shopping center. Which types of stormwater management systems should the architect recommend? (Choose the three that apply.) (A) An underground cistern, fed from rainwater collection on the shopping center roof (B) Porous asphalt in the parking lot (C) Foundation drainage around building perimeter (D) A construction stormwater wetland planted with native vegetation (E) Onsite graywater treatment (F) Diversion ditches between adjacent rows of parking spaces

(A), (B), (F) Runoff from impervious surfaces, such as roofs or paving, can cause erosion and flooding. The underground cistern will instead capture the rainwater, and porous paving will allow the water to seep back into the soil Diversion ditches will collect any additional runoff from the parking area and divert it away from the built areas Foundation drainage and graywater treatment options are not related to stormwater management and a constructed wetland is not appropriate for a small commercial site in a cold climate

To determine the regulations that pertain to a planned development around a wetlands area, which of the following should the architect investigate? (Choose the three that apply.) (A) local governmental rules (B) local building codes (C) state governmental rules (D) US Army Corps of Engineers regulations (E) development covenants (F) zoning ordinances

(A), (C), (D)

A small building is being designed for a site in Minnesota. If it is desired to minimize reliance on mechanical systems, which of the following design strategies should be incorporated? (Choose the three that apply.) (A) maximize south-facing windows (B) incorporate high ceilings (C) design a compact form (D) use dark colors for the building exterior (E) minimize interior thermal mass (F) use evergreen trees on the south side of the building

(A), (C), (D) Minnesota has a cool to cold climate. South-facing windows are good for passive solar heat gain. A compact form minimizes the surface area to reduce heat loss during the winter. Dark colors absorb more solar radiation than light colors Option (B) is incorrect because, used with natural ventilation, high ceilings are more appropriate for a hot-humid climate Option (E) is incorrect because thermal mass should be maximized to take advantage of solar heat gain. In a cold climate it is desirable to have as much mass as possible to absorb and store the heat gathered during daylight hours for use at night Option (F) is incorrect because deciduous trees, not evergreen, are desirable on the south side of the building to let in the sun during the winter

The energy cost budget method, as defined in ASHRAE/IESNA Standard 90.1, is recommended for which of the following types of buildings? (Choose the four that apply.) (A) building that utilizes passive solar heating (B) convenience store operating 24 hours a day, 7 days a week (C) office building powered with photovoltaic panels (D) retail store designed as a zero-net energy structure (E) building with no mechanical system (F) two-story apartment building

(A), (C), (D), (E)

Which of the following statements about sawn lumber joists in wood-framed buildings are true? (Choose the four that apply.) (A) Joists often have a nominal width of 2" (B) Joists often have a nominal depth of 6" to 28" (C) For heavy loads, joists may have a nominal width of 3" to 4" (D) Double joists must be provided under partitions parallel to the joists (E) Joists are often spaced at 12" or 16" on center (F) Joists rarely span more than 10'

(A), (C), (D), (E) Option (B) is false, lumber joists often have nominal depths of 6" to 14". The depth of 28" is not included in the table of standard sizes for sawn lumber joists Option (F) is false, lumber joists often span 15' to 20', and sometimes more

Which of the following are true about life-cycle cost analysis (LCCA)? (Choose the four that apply.) (A) Higher up-front costs often lead to lower operational costs (B) Cost presented in the analysis are the only factors that should be considered when making a decision about a material or system (C) It is economical to complete a LCCA if two items have similar installation costs but differing operational costs or projected lifespans (D) LCCA evaluates all costs based upon their present value (E) A LCCA is similar to a life-cycle assesment (F) Salvage fees may be included in an LCCA

(A), (C), (D), (F) LCCA allows designers and owners to evaluate the total cost of a product or system over its useful life. This facilitates an objective comparison between two or more options based upon a number of factors, including their initial purchase price, long-term maintenance costs, replacement costs over time, and any portion of the investment that may be recouped through salvage fees. Of course, this analysis is not the only factor that should be considered when making decisions about what to include in a building's design, but it does permit a comprehensive look at the total cost to the owner and that decision will incur. An item that looks like a bargain in terms of its present day purchase price may have steep operational costs or a short lifespan, requiring replacement in just a few years. Life-cycle costing takes all of these factors into consideration and converts all of the costs to present-day values so that they can be compared effectively. An LCA (Life-cycle assessment) is a method of evaluating the environmental impact of using a material or product and is not the same as an LCCA

Air barriers are designed to stop infiltration and ex-filtration caused by which of the following? (Choose the four that apply.) (A) stack effect (B) vapor pressure (C) heating, ventilating, and air conditioning (HVAC) fan pressure (D) wind pressure (E) temperature differentials (F) exhaust vents

(A), (C), (D), (F) Wind pressure, stack pressure, and HVAC fan pressure all can influence infiltration and ex-filtration rates. Vapor pressure does not cause air movement; rather, vapor pressure is a movement of moisture. Temperature differentials have little effect on infiltration or ex-filtration

Of the below listed agencies, which are responsible for material ratings? (Choose the three that apply.) (A) ANSI (B) ADA (C) ASTM (D) ICC (E) UL

(A), (C), (E) ASTM, ANSI, and UL are all responsible for material and/or product ratings ADA and ICC are code and legislative related items

Which of the following factors affect a building's response to an earthquake? (Choose the four that apply.) (A) soil type (B) local temperature (C) building form (D) building orientation (E) building weight (F) structural materials and system

(A), (C), (E), (F)

Electrical operating costs in a single-tenant commercial building can be minimized by using which of the following techniques? (Choose the four that apply.) (A) daylighting (B) indirect lighting (C) load control (D) multiple metering (E) occupancy sensors (F) LED lamps

(A), (C), (E), (F) Daylighting can be used to reduce electrical lighting requirements Load control is a way of avoiding peak-demand electrical charges by automatically or manually shutting off nonessential electrical loads before the peak demand is reached Occupancy sensors automatically turn light on or off depending on whether a space or room is occupied LED lamps can minimize power required for lighting compared with compact fluorescent or incandescent lamps Indirect lighting alone probably would not reduce power use; in fact, it may increase power because more wattage might be needed to achieve the required lighting level than would be needed with direct or task lighting Multiple metering is used for multi-tenant spaces only and would not result in an overall cost savings for a commercial building

Which of the following have a significant effect on heat gain? (Choose the four that apply.) (A) sunlight (B) humidity (C) motors and equipment (D) air movement (E) people (F) fluorescent lighting

(A), (C), (E), (F) Humidity and air movement do not affect heat gain. Although the effect of motors and equipment, people, and lighting may vary in different types of occupancies, they all produce heat

In case of a fire, which of these can be activated by building occupants? (Choose the three that apply.) (A) fire extinguishers (B) dry standpipes (C) wet standpipes (D) fusible links (E) annunciators (F) two-way communication devices

(A), (C), (F)

Which of the following affect a wall's fire-resistance rating? (Choose the two that apply.) (A) Material of the wall (B) Internal temperature of the wall (C) Dew point of the wall (D) Wall thickness

(A), (D) A wall's fire resistance rating is affected by the materials it's composed of and the thickness (and/or arrangement) of those materials

A new, one-story daycare center is being planned in a hilly, suburban location in a hot/humid climate. The prevailing winds are from the south. The client wants to use passive cooling. Which of the following strategies should the architect recommend? (Choose the three that apply.) (A) The building should be located at the top of the slope (B) The building should be elongated along the north-south axis (C) The playground should be located north of the building (D) The building's main corridor should include operable transoms above the doors (E) The building should include large openings on the north and south sides (F) A line of shrubs should be planted west of the playground

(A), (D), (E) Buildings in a hot/humid climate should be located at the top of a slope to catch cooling breezes Large openings on the north and south sides and operable transoms in the corridor will allow for cross-ventilation A north-south axis would actually limit the opportunities for cross-ventilation and expose the building to the hot afternoon sun Placing the playground north of the building would provide limited shading but would block cooling breezes from reaching the playground Planting shrubs west of the playground may provide visual interest but would not be effective in blocking the hot afternoon sun

To determine the required fire resistance for an exterior, nonbearing wall, which factors must an architect consider? (Choose the three that apply.) (A) construction type (B) size of openings in the wall (C) fire resistance rating of wall materials (D) percentage of opening area to total area (E) occupancy (F) distance from property lines

(A), (E), (F) Once the architect uses an IBC table to determine the hourly fire-resistance rating, the required fire-resistance rating of windows and doors can be determined. For example, fire protection-rated glazing is not even permitted in a 2-hr rated wall, and doors must have a 90 min fire resistance rating. However, fire-resistance-rated glazing can be used if it has a 2-hr rating for the maximum size tested. The required fire-resistance rating of the wall must first be determined before selecting building materials and assemblies to meet the rating. The percentage of opening area has nothing to do with fire resistance of an exterior wall. The fire-resistance rating depends on the construction type, occupancy, and separation of the building from the property line

The type of lockset that is most secure is a (A) unit lock (B) cylindrical lock (C) rim lock (D) mortise lock

(B)

Rank the below materials in order from highest (top) to lowest (bottom) R-Value per inch (A) Polyurethane rigid panel Fiberglass loose fill High density fiberglass batts Single glass pane Straw bale (B) Polyurethane rigid panel High density fiberglass batts Fiberglass loose fill Straw bale Single glass pane (C) High density fiberglass batts Polyurethane rigid panel Fiberglass loose fill Straw bale Single glass pane (D) Fiberglass loose fill High density fiberglass batts Polyurethane rigid panel Single glass pane Straw bale

(B) Polyurethane rigid panel = R-7-8 /inch High-density fiberglass batts = R-3.6-5 /inch Fiberglass loose fill = R-2.4-3.6 /inch Straw bale = R-1.45 /inch Single glass pane = R-0.14 /inch

Which of the following answers correctly lists the uses in order of lowest (at top) to highest (at bottom) sq. ft. per occupant? (A) Office (Business area) Casino floor Dormitory Aircraft hangar Dwelling (Residential) (B) Dwelling (Residential) Dormitory Aircraft hangar Casino floor Office (Business area) (C) Casino floor Dormitory Office (Business area) Dwelling (Residential) Aircraft hangar (D) Aircraft hangar Dwelling (Residential) Office (Business area) Dormitory Casino floor

(B) Square footage per occupant is also known as occupant load. Occupant load for different functions is identified on table 1004.1.2 of the IBC. Casino floor (Assembly gaming floors) = 11 sq ft./ occupant Dormitory = 50 sq. ft. / occupant Office (Business area) = 100 sq. ft. / occupant Dwelling (Residential) = 200 sq. ft. / occupant Aircraft hangar = 500 sq. ft. / occupant Occupant load can often be confusing because higher square footage per occupant means the space can accommodate fewer occupants

An architect has designed a bakery/coffee shop in Pittsburgh, Pennsylvania. The business is such a success that three years later, the owner decides to allow her sister, who lives in Durham, North Carolina, to open a similar store using the same name, logos, and decor. After consultation with the architect, they determine that a building of similar size and design will be appropriate for the Durham location. The sisters ask the architect for a rough projection of the cost to build the new bakery/coffee shop based upon the construction cost of the Pittsburgh store. The location factor for Pittsburgh is 100. The cost factor for Durham is 75. The inflation rate has been 2% per year since the first building was built. The original construction cost was $300,000. Based on the information given, approximately how much will it cost to construct the new building? (A) $225,000 (B) $239,000 (C) $306,000 (D) $318,000

(B) $239,000

A three-story speculative office building has a footprint of 6,724 ft2. The floors are equal in size. The building is sited on a 1.5 acre parcel. The floor area ratio is approximately (A) .1:1 (B) .3:1 (C) .6:1 (D) 1:1

(B) .3:1 The floor area ratio expresses the relationship between the square footage of the building and the area of the site on which it is constructed. Zoning ordinances often set limits on the maximum floor area ratio allowed within a region as a means of controlling development density Area building = = (3 floors) (6,724 ft2) = 20,172 ft2 Area site = = (1.5 acre) (43,560 ft2/acre) = 65,340 ft2 FAR = = (Area building) / (Area site) = (20,172 ft2) / (65,340 ft2) = .3087

What is the minimum distance between distance between two elevators on either side of an elevator lobby? (A) 8' (B) 10' (C) 12' (D) 14'

(B) 10' The minimum width of a lobby should be 10 feet

A room 15ft by 20ft with a 9ft ceiling has a carpeted floor with a 44oz carpet on pad (a=0.40), gypsum board walls, and a gypsum board ceiling (a=0.05). What will be the noise reduction achieved by directly attaching acoustical tile with an NRC of 0.70 to the ceiling? (A) 1.5 dB (B) 3.4 dB (C) 5.1 dB (D) 11 dB

(B) 3.4 dB Find the total absorption of the original room A1 = Afloor + Aceiling + Awall1 + ... + Awall4 = (15ft)(20ft)(0.40) + (15ft)(20ft)(0.05) + ... = 167 sabins Find the total absorption of the new room A2 = Afloor + Aceiling + Awall1 + ... + Awall4 = (15ft)(20ft)(0.40) + (15ft)(20ft)(0.70) + ... = 362 sabins Calculate the noise reduction NR = = 10log(A2/A1) = 10log(362/167) = 3.359 dB

A wall is 9ft high and 15 ft long and contains a 7ft by 3ft door. The transmission loss of the wall alone is 54 dB. The transmission loss of the door with full perimeter seals is 29 dB. What is the combined transmission loss of the wall and door? (A) 25 dB (B) 37 dB (C) 56 dB (D) 83 dB

(B) 37 dB Total Area of Door and Wall = (9)(15) = 135 ft2 Total Area of Door = (7)(3) = 21 ft2 Total Area of Wall = 135-21 = 114 ft2 Transmission Loss Wall = 10^(-54/10) = 10^(-5.4) Transmission Loss Door = 10^(-29/10) = 10^(-2.9) Transmission Loss Composite Door and Wall = 10log(Area Total / Sum(ts)) = 10log(135ft2 / (10^(-5.4)x(114ft2) + 10^(-2.9)x(21ft2)) = 37.01 dB

A room in an office contains eight machines, each of which produces 73 dB of sound. What is the overall sound level? (A) 78 dB (B) 82 dB (C) 87 dB (D) 92 dB

(B) 82 dB Use the following equation where n is the number of sources of identical value Total = 73 dB + 10log(n) = 73 dB + 10log(8) = 82 dB

What is laitance? (A) A high-strength layer of fine particles that floats to the surface of wet concrete (B) A high-strength layer of large particles that sinks to the bottom of wet concrete (C) A low-strength layer of fine particles that floats to the surface of wet concrete (D) A low-strength layer of large particles that sinks to the bottom of wet concrete

(B) A high-strength layer of large particles that sinks to the bottom of wet concrete The fine aggregates separate from mix and float to the top due to too much water

An architect is designing a warehouse in Maine that will require a sprinkler system. What kind of sprinkler system would make the most sense? (A) Pre-action (B) Dry Pipe (C) Wet Pipe (D) Deluge

(B) Dry Pipe Dry pipe sprinklers are similar to pre-action systems as they use pressurized air in the pipe which exits before water escapes. This causes a minute delay in water discharge but is ideal for buildings with low temperatures so the pipes do not freeze. These fire sprinkler systems have a fast opening tool to get rid of the air and speed up the flow of water. Warehouses located in the North are a good example of what buildings should use dry pipe sprinklers

Which organization would most likely provide financial incentive for the remediation of a brownfield site? (A) ASHRAE (B) EPA (C) ASTM (D) ICC (E) ANSI (F) LEED

(B) EPA A brownfield is a former industrial or commercial site where future use is affected by real or perceived environmental contamination The EPA provides grants for brownfield site remediation

Which of the following tree arrangements is best suited for optimal solar energy utilization and control for a house in the northeast region of the United States? (A) Evergreens on the east and south; deciduous on the west (B) Evergreens on the west and north; deciduous on the south (C) Evergreens on the west and north; no trees on the south (D) Evergreens on the north and south; deciduous on the east and west

(B) Evergreens on the west and north; deciduous on the south Since the site is in the northeast region of the United States, evergreen trees to the north and west will help to block winter winds, as well as the heat gain from western sunlight in the summer. Deciduous trees to the south allow winter sun, when it is desired for passive heat gain, but block summer sun, providing valuable shade

What is the principal cause of structural damage, injury, and loss of life during an earthquake? (A) Gale force winds (B) Ground shaking (C) Sinkholes (D) Sublimation

(B) Ground shaking The principal cause of structural damage, injury, and loss of life during an earthquake is ground shaking

For which of the following building types does the structural system represent the smallest percentage of the total construction cost? (A) Warehouse (B) Hospital (C) Parking Garage (D) Shopping mall

(B) Hospital Warehouses and parking garages are primarily structural in nature with little finish work. The structural costs for these projects would be the greatest percentage of the overall project costs Shopping malls and hospitals are both public spaces with high-end finishes. However, hospitals have many more requirements and are considered 'high hazard' which cause the cost of non-structural items to be higher than a shopping mall

The sole purpose of programming in architecture is to: (A) Collect data (B) Identify the problem (C) Create goals for the project (D) Generate solutions

(B) Identify the problem The sole purpose of creating an architectural program is to identify the problem Collecting data and creating goals are part of the programming process, but not its sole purpose Solutions are generated in the design phase which comes after the program has been completed

What is ideal for buildings in cold climates? (A) Minimizing exposed surface area to encourage heat loss (B) Minimizing exposed surface area to discourage heat loss (C) Maximizing exposed surface area to encourage heat loss (D) Maximizing exposed surface area to discourage heat loss

(B) Minimizing exposed surface area to discourage heat loss In cold climates it is ideal to minimize exposed surface area to discourage heat loss

What is the term for a pitted area sometimes found in cedar wood? (A) Knot (B) Peck (C) Shake (D) Wane

(B) Peck A pitted area sometimes found in cedar or cypress wood is called a peck

To improve indoor air quality, what interior materials should be specified? (A) LEED certified products (B) Products with low VOCs (C) Energy-star products (D) Products that are sustainably harvested

(B) Products with low VOCs VOCs or (volatile organic compounds) are organic chemicals that have a high vapor pressure at ordinary room temperature. Their high vapor pressure results from a low boiling point, which causes large numbers of molecules to evaporate or sublimate from the liquid or solid form of the compound and enter the surrounding air Since many people spend much of their time indoors, long-term exposure to VOCs in the indoor environment can contribute to sick building syndrome. In offices, VOC results from new furnishings, wall coverings, and office equipment such as photocopy machines, which can off-gas VOCs into the air. Good ventilation and air-conditioning systems are helpful at reducing VOCs in the indoor environment

A material supplier states that adding a certain product to a wall assembly in a critical acoustical situation will increase the noise reduction (STC) between two spaces by 3 dB. What is the most appropriate response? (A) Determine the additional cost and then decide whether or not to use the product (B) Thank the supplier for stopping by but explain that the architectural firm probably will not be using the product because that amount of noise reduction does not make it worth the effort or cost (C) Specify the product as long as it does not affect the design or construction cost by more than 5% (D) Inquire whether some modification can be made to the product to increase its rating to 6 dB and say that then the architectural firm might consider it

(B) Thank the supplier for stopping by but explain that the architectural firm probably will not be using the product because that amount of noise reduction does not make it worth the effort or cost Because a change in 3 dB is considered "just perceptible," it would probably be better not to use the material regardless of how low the added cost was Trying to modify the material to 6 dB would probably not be worth the trouble, either

What is an exit discharge? (A) This distance that occupants have to travel to reach an exit (B) The route between a building exit and a safe public way (C) Direct access to a public way (D) The portion of a means of egress system that leads from any occupied portion of a building or structure to an exit

(B) The route between a building exit and a safe public way Exit discharge is defined in the building code as "that portion of a means of egress system between the termination of an exit and a public way."

IBC contains requirements for the ventilation of attics and concealed roof spaces for which of the following reasons? (A) To prevent heat transfer from the building into the spaces (B) To prevent moisture accumulation and condensation in the spaces (C) To eliminate the need for installation of an air barrier in the spaces (D) To exhaust fire or smoke from the spaces

(B) To prevent moisture accumulation and condensation in the spaces When building materials are enclosed on both the exterior and interior side of a space, the concealed space - whether an attic or rafter space - becomes susceptible to moisture intrusion. Ventilation paths allow moist air to be exhausted from a concealed space The other options while important to consider when designing a building, are not relevant to this code requirement

Greywater is water that has originated from what source? (A) Captured rainwater (B) Water from building fixtures other than water closets (C) Any water that has been used only once in a building (D) Water from water closets

(B) Water from building fixtures other than water closets Greywater is any household wastewater with the exception of wastewater from toilets (water closets), which is known as blackwater

Ceramic mosaic tile in a public shower room is best installed over a (A) water-resistant gypsum board (B) bed of portland cement mortar (C) concrete block wall coated with a waterproofing membrane (D) rigid cement composition board made for this purpose

(B) bed of portland cement mortar A full bed of portland cement mortar offers the best durability and water resistance for high-use, wet areas

For a three-story department store, the most important variable for selecting an elevator would be its (A) speed (B) capacity (C) control method (D) machine room location

(B) capacity A department store requires elevators with capacity. For a three-story building, speed is not critical, and the control method and machine room location would be secondary considerations It is important to understand the different elevator control methods available. These include the single automatic, selective collective, and group automatic: 1 - A single automatic system answers only one call at a time, and the user has exclusive control of the car until the trip is complete. 2 - With a selective collective system, the elevator answers all calls in one direction and then reverses direction and answers all calls in the opposite direction 3 - With a group automatic system, a computer controls two or more elevators and dispatches and operates all the elevators in the most efficient manner possible

The stringers of prefabricated steel utility stairs are normally constructed of (A) angle iron (B) channel sections (C) steel plate (D) tube sections

(B) channel sections Although any of the listed forms can be used, the stringers are normally constructed of steel channel sections with the flanges turned away from the stair. The steel treads and risers are welded to the webs of the sections

In a large single-tenant building, a LAN system would most commonly serve (A) building automation (B) computers (C) security (D) telephones

(B) computers LAN is an acronym for local area network, which is a system of individual computers, computer servers, and wired or wireless connections that allows all the users in an individual building or complex of buildings to share data on a nonpublic network

Which of the following flooring types has the highest resilience? (A) asphalt (B) cork (C) linoleum (D) vinyl composition

(B) cork Cork is a very resilient material. Its resilience is similar to that of rubber tile Asphalt tile, which is seldom used, has the lowest resilience. Linoleum and vinyl composition tile have low to moderate resilience

Which of the following principles is best exemplified by the "whispering arch" at Union Station in St. Louis, Missouri? (A) focusing (B) creep (C) diffusion (D) specular reflection

(B) creep

What is especially important in designing roads for drainage? (A) curbs and gutters (B) crown (C) catch basin (D) superelevation

(B) crown

The design occupant load of a hotel ballroom is determined by (A) drawing proposed layouts of the room for receptions, meetings, and dining, and figuring out how many chairs will fit into the space in each scenario (B) dividing the total square footage of the space by a factor that allocates space for each person (C) multiplying the number of people expected to use the space by an assumed weight per person (D) asking the conference services manager how many peopl must be accommodated in the space

(B) dividing the total square footage of the space by a factor that allocates space for each person

The greatest degree of protection from cold winter winds can be achieved with (A) airlocks (B) earth sheltering (C) green roofs (D) landscaping

(B) earth sheltering Earth sheltering would offer the greatest degree of protection from cold winter winds Airlocks only protect door openings Green roofs are primarily used to protect against solar radiation and to reduce runoff Landscaping can reduce the negative effects of wind, but not as well as solid earth

During design development for a small corporate headquarters building, the client informs the architect that, due to poor sales in the previous quarter, the budget for the project must be reduced. Which of the following should the architect ignore in order to reduce the initial construction costs? (A) reducing the number of pieces in the details (B) eliminating high maintenance finishes (C) suggesting changes that would make custom finishes closer to industry standards (D) reducing the number of different details involved in the project

(B) eliminating high maintenance finishes Using industry-standard details, reducing the number of components in the construction assembly, and reducing the number of unique details all would help reduce construction costs Although life-cycle costs are a consideration in a corporate headquarters (presumably owner-occupied and maintained), the problem refers to initial construction costs

Which of the following is a good strategy for minimizing light pollution on a site? (A) eliminate parking lot lighting (B) employ light bollards instead of pole-mounted luminaires (C) specify high-reflectance surfaces (D) reduce light trespass at night from inside buildings on the site

(B) employ light bollards instead of pole-mounted luminaires

Which person or entity generally has the final approval in determining where exit signs are located in a building? (A) International Building Code (IBC) (B) field inspector (C) Occupational Safety and Health Administration (OSHA) (D) city code plan checker

(B) field inspector Although the IBC gives specific requirements for exit sign placement and exceptions for where they are not required, the local building inspector usually has the final say in where they are required. This is because only when the actual conditions of construction are observed can an assessment be made of how well or even if the exit signs are visible. Even though the placement of exit signs by the architect and electrical engineer may meet the letter of the IBC, in the field there may be areas where the signs are not visible. OSHA only gives testing requirements for exit signs. The city code plan checker may have an opinion about the exit sign location, but the final responsibility rests with the building department

A row of trees of moderate density will reduce the wind velocity on the leeward side by about 30-40% up to about (A) three times the height of the trees (B) five times the height of the trees (C) seven times the height of the trees (D) nine times the height of the trees

(B) five times the height of the trees For a row of trees of moderate density, wind speed is reduced 30-40% on the leeward side up to a distance of about five times the tree height

A structural member consisting of a 2x12, steel plate, and another 2x12 sandwiched together is known as a (A) composite beam (B) flitch beam (C) built-up beam (D) sistered beam

(B) flitch beam A flitch beam combines wood and steel into one member with load-carrying capacity far exceeding that of wood alone. Flitch beams are sometimes referred to as sandwich beams

What type of elevator should be specified for a 40-story office building? (A) hydraulic (B) gearless traction (C) geared traction (D) electric

(B) gearless traction

What type of elevator should be specified for a 40-story office building? (A) hydraulic (B) gearless traction (C) geared traction (D) electric

(B) gearless traction Gearless traction elevators can travel at the highest speeds of the types listed, and to accommodate the rush of people entering and exiting the building in the morning, at lunch time, and in the evening, the quick cycles will be a necessity Gearless traction and geared traction are two different types of electric elevators. Both operate on DC current. The geared traction elevator travels at slower speeds but offers many options for adjusting the speed to suit the building conditions Hydraulic elevators are lifted by a ram, which must be sunk into the ground the same distance as the height of the elevator's path of travel. Therefore, they are used only in low-rise buildings (generally less than six stories). They travel much more slowly than electric elevators and are better used for freight or for low-occupancy passenger elevators where speed is not an issue

A small office is being planned for a new site. What feature will have the most influence on the solar design in the overall building configuration? (A) existing landscape (B) geographic location of the site (C) topography (D) vehicular access

(B) geographic location of the site Placement of solar features is most affected by where in the world the building is located. Passive solar features should be placed facing the side of the building that will get the most sun

Which of these is the most important concern in relation to a private water supply? (A) fixture pressure (B) hardness (C) friction loss (D) yield

(B) hardness Hardness affects the quality and taste of water as well as the longevity of the plumbing system. Very hard water can deposit minerals that build up in pipes and on plumbing fixtures. The taste may be objectionable, requiring installation of a water-softening or filtration system

The architect can minimize the heat island effect of impervious site paving by selecting a material with (A) low albedo (B) high albedo (C) low conductivity (D) high conductivity

(B) high albedo Albedo is a measure of how much of the radiant energy that a surface receives is reflected rather than absorbed. It is expresses as a fraction from zero to one. A surface with a high albedo will reflect more energy Selecting a material with a low conductivity will not have as much of an effect on the site's temperature

A retention pond manages stormwater runoff by (A) slowing it and allowing sediments to settle while letting the water seep into the ground (B) holding the excess until it can discharge at a controlled rate into the storm sewer system (C) preventing it from contaminating other portions of the site (D) retaining it until it can seep into the ground

(B) holding the excess until it can discharge at a controlled rate into the storm sewer system A retention pond, also called a holding pond or catch basin, prevents excessive stormwater runoff on a site from overloading the storm sewer system by temporarily holding the water and releasing it at a controlled rate Construction that is designed to allow sediment to settle while water drains into the ground is called a bioswale Construction designed to retain stormwater until it can seep into the ground is called an infiltration basin The term catch basin sometimes also refers to a storm drainage structure that is designed to collect grit and trash while allowing the stormwater to flow out of the drainage pipes

Which would be the best location for a transformer for a large school building? (A) on the power pole serving the building (B) in a separate room near the exterior wall (C) outside, on a transformer pad close to the main switchgear (D) in a protective shed where power from the utility company enters the property

(B) in a separate room near the exterior wall A transformer vault near the exterior wall would be the best choice for protection, ventilation, and ease of installation and removal. A large school building would require high voltage service from the utility company and step-down transformers provided to the owner This type of transformer could not be installed on a pole. Although the transformer could be placed on a pad outside the building, this would leave it exposed to possible vandalism and might present a danger to the students

Which of the following types of lamps provides the best color rendition of skin tones? (A) cool-white fluorescent (B) incandescent (C) mercury vapor (D) metal halide

(B) incandescent An incandescent lamp has a high color rendering index (CRI) and low color temperature, with a predominance of light in the red range. These characteristics give a complimentary rendering to skin tones A cool white fluorescent lamp has more blue and green light in its spectral distribution and makes skin tones appear more gray and washed out Both mercury vapor and metal halide lamps have poor color rendering indices and are not appropriate for lighting areas where skin tone rendition is important

An architect is evaluating the attributes of a particular building product as they relate to sustainability. The architect would most likely use a(n) (A) environmental impact study (B) life-cycle assessment (C) impact assessment (D) matrix comparison chart

(B) life-cycle assessment A life-cycle assessment evaluates the environmental impact of a particular material over its entire useful life, including disposal. It could be used to compare the impacts of two or more materials so the architect could select the most sustainable one An environmental impact study (EIS), is used to evaluate the impact of a development on the environment An impact assessment is one phase of a life-cycle assessment There is no sustainability method by the name "matrix comparison chart"

Which of the following buildings will be a good application of demand-controlled ventilation (DCV) technology? (A) pet store (B) locker room (C) bowling alley (D) dry cleaner

(B) locker room

Which of the following spaces should be acoustically "live" (Choose the two that apply.) (A) talk radio recording studio (B) cathedral featuring an antique German pipe organ (C) opera house (D) community playhouse often used for poetry recitation (E) university lecture hall (F) movie theater

(B), (C)

The moment-resisting frame is an appropriate system to use for resisting wind forces in (A) low-rise buildings only (B) low-rise buildings and high-rise buildings of 30 stories or less (C) high-rise buildings of approximately 50 stories (D) high-rise buildings of any height

(B) low-rise buildings and high-rise buildings of 30 stories or less The moment-resisting frame has rigid moment-resisting connections between beams and columns. It is an appropriate system to use for resisting lateral forces, not just in low-rise buildings, but also in high-rise buildings with 30 stories or less For taller buildings, other systems should be considered, or some additional type of bracing such as X or chevron bracing

For a large building being planned with a two-level basement used for meeting rooms, which of these water- related soil problems would be the most important to solve? (A) uplift pressure on the lowest slab (B) moisture penetration caused by hydrostatic pressure (C) deterioration of foundation insulation (D) reduced load-carrying capacity of the soil

(B) moisture penetration caused by hydrostatic pressure Option A would not cause as many problems as Option B, but still needs to be considered Option C is unlikely as foundation insulation could easily be selected to avoid deterioration problems Option D is unlikely because a large building would probably utilize piers or caissons for the foundation, so the load-carrying capacity of the soil would not be as critical

Storm runoff is best minimized with the use of (A) cisterns (B) pervious paving (C) riprap (D) silt fences

(B) pervious paving Pervious paving allows rainwater to soak into the ground while providing support for parking or other outdoor activities Cisterns are designed for holding rainwater for further use, not to minimize the runoff Riprap is stone reinforcement for the banks of rivers or lakes Silt fences are used to prevent erosion and sediment runoff during construction

A photoelectric detector will warn of a fire when the fire reaches the (A) incipient stage (B) smoldering stage (C) flame stage (D) heat stage

(B) smoldering stage

Which of the following defects would most affect a wood joist's ability to resist horizontal shear? (A) knot (B) split (C) wane (D) warp

(B) split A split is a separation of the wood fibers along the grain that extends through the piece of lumber Because the value of horizontal shear depends on the integrity of the wood along its grain, any break would reduce the ability of the wood to resist horizontal shear

According to model codes, which of the following are considered parts of the means of egress? (Choose the three that apply.) (A) common path of travel (B) exit (C) exit access (D) exit discharge (E) public way (F) travel distance

(B), (C), (D)

Which of the following woods must be treated for resistance to decay when used in an exterior application? (A) cedar (B) spruce (C) redwood (D) cypress

(B) spruce Spruce trees are not inherently resistant to decay, but the heartwood of cedar, redwood, and cypress all possess a natural resistance that makes them a good choice for use in exterior applications such as siding, shingles or shakes, and decking. The outer rings of these species do not possess the same decaying resistant qualities, so it is important to specify that only heartwood may be used when decay resistance is of concern. These woods are considerably more expensive than other species that are more commonly used for construction such as pine, hemlock, fir, and spruce Species that are not inherently decay resistant must be chemically treated to protect them from rotting when exposed to moisture. Waterborne salts are pressure-impregnated into the wood to prevent decay, often for as long as 30 years. After a waiting period, salt-treated wood can be painted or stained

An architect is designing a 10-story building and is analyzing the options for the cladding system. The architect has determined that the building will utilize a prefabricated system with a variety of glass vision and spandrel panels. Although the structural grid of the building is very regular, to add visual interest to the facade, the panels will be a variety of sizes. Which type of cladding system will be the best desirable choice for this application? (A) curtain wall stick system (B) substrate mounted composite metal and glass panels (C) curtain wall panel system (D) metallic finish EIFS system

(B) substrate mounted composite metal and glass panels

What is the primary purpose of the voids in a cored slab (A) to allow electrical services to be concealed in the slab (B) to make a more efficient load-carrying member (C) to make erection easier (D) to minimize weight

(B) to make a more efficient load-carrying member As with any beam, the deeper the member, the more efficient the beam. Using a cored slab rather than a solid slab allows the depth to be increased without increasing the weight in the center of the beam where it is not needed. Option (A) and (D) are partially correct, but option (B) is the best choice

What is the intent of defensible space? (A) to design spaces so that law enforcement can better survey and control a neighborhood (B) to protect private property owners from vandalism and crime (C) to allow residents to control the areas around their homes (D) to mix income levels within housing facilities

(B) to protect private property owners from vandalism and crime

An office in a non-sprinklered building has an occupant load of 290. In a non-sprinklered building, the IBC requires a minimum exit width of 0.2 in per person based on the occupant load. Which of the following exit door combinations would most minimally meet the exit width required by the IBC? (A) a pair of 30 in entry doors and a 36 in door remotely located (B) two 36 in doors in opposite sides of the building (C) three 32 in doors remotely located (D) three 36 in doors remotely located from each other

(B) two 36 in doors in opposite sides of the building Find the total exit width required: Width = = (occupant load) (0.2 in/person) = (290 people) (0.2 in/person) = 58 inches Any exit door must provide a clear width of at least 32 in, so option (A) and (C) cannot be correct because their clear widths would be less than 32 inches Three 36 in doors would be acceptable, but the question asks for the minimally acceptable solution, which is two 36 in doors; this would provide approximately 66 in of width (considering the clear width of the door to be open from the doorstop to the face of the door when open)

The roof of a building is generally subject to what type of wind pressure? (A) direct (positive) pressure (B) uplift, or suction (C) drag forces (D) combination of direct (positive) pressure and suction

(B) uplift, or suction This is why roofs are often blown off during hurricanes and tornados

When determining the massing of a building during preliminary design, what should the architect research first? (A) view ordinances (B) zoning regulations (C) utility easements (D) municipal master plan

(B) zoning regulations

Which of the follow would affect the life-cycle cost of an HVAC system? (Choose the four that apply.) (A) Property costs (B) Investment costs (C) Maintenance & repair costs (D) Amortization costs (E) Operating costs (F) Variable costs

(B), (C), (D), (E) Life-cycle cost in equation form is: LCC = IC + MC + AC + OC IC = investment cost (first cost minus salvage) MC = maintenance & repair cost AC = amortization cost (replacement) OC = operating cost (including labor and energy)

Variables that must be considered when designing for daylighting include which of the following? (Choose the four that apply.) (A) Longitude of sight (B) glass transmittance (C) height of the head of the glass (D) height of the space above ground level (E) outdoor surface reflection (F) room surface reflection

(B), (C), (E), (F) Variables that affect daylighting include the brightness of the sky (which is affected by solar altitude, cloud conditions, and time of day), the area of the glass, the height of the head of the glass, the transmittance of the glass, the reflectance of surfaces within the room and nearby outdoor surfaces, and obstructions such as overhangs and trees

Which building planning strategies are most appropriate to use for a building located in Omaha, NE? (Choose the two that apply.) (A) Use coniferous trees for shade (B) Use deciduous trees for shade (C) Minimize north-south exposure (D) Minimize east-west exposure (E) Provide permanent wind protection

(B), (D) Omaha, Nebraska is located in a temperate climate Buildings in temperate climates should: + minimize east and west exposures + balance solar heat gain in the + summer/winter with deciduous trees + encourage air movement in summer + protect against wind in winter

In which conditions is wood strongest? (Choose the two that apply.) (A) Forces perpendicular to grain (B) Forces parallel to grain (C) Shear parallel to grain (D) Shear perpendicular to grain

(B), (D) Wood is strongest against forces applied parallel to the grain and when shear is perpendicular to the grain

In site design, an Architect must create positive drainage away from which of the following? (A) Public streets (B) On-site parking areas (C) Bodies of water (D) Walkways (E) Buildings

(B), (D), (E) An architect must create positive drainage away from: + parking areas + walkways + buildings + The architect may or may not be required to drain away from bodies of water. Drainage for public streets is not typically a part of the Architect's responsibility

An Architect is preparing a cost estimate for a new building. Which resources can be used to aid in this cost estimate? (Choose the three that apply.) (A) The AIA (B) Architects' historical records (C) Code enforcement agencies (D) Commercially published data (E) General contractors (F) Plan rooms

(B), (D), (E) Common resources for helping Architects create cost estimates are: + Historical records of previous projects + Commercially published cost data + General contractors or other cost estimating experts

Which of the following can be applied as a protective coating to steel in order to provide fire resistance? (Choose the three that apply.) (A) Caulk (B) Concrete (C) Glulam members (D) Gypsum wallboard (E) Plaster (F) Water vapor

(B), (D), (E) Steel can be coated with concrete, (fire rated) gypsum wallboard, and plaster to provide fire resistance. It is also common for steel needing fire-resistance to have an intumescent coating applied

The zonal cavity method of calculating average illumination on the work surface for a given number of luminaires depends on which of the following variables? (Choose the four that apply.) (A) angle of light (B) dirt accumulation (C) efficacy of the lamp (D) lumens per lamp (E) room size (F) wall reflectance

(B), (D), (E), (F) The zonal cavity method is used to calculate the total number of luminaires required to achieve the desired average illumination on the work surface, which is assumed to be 30" above the floor. The variables considered in the calculation include the level of illumination desired, the area of the room, the number of lamps in each luminaire, the lumen output of each lamp, the coefficient of utilization, the light loss factor, wall reflectance, and the gradual loss of light due to dirt accumulation The coefficient of utilization (CU) is a measure of the efficiency of a particular luminaire in outputting light and is based on the design of the luminaire itself, the reflectance of the room, and the size of the room. The light loss factor represents a degradation of ideal light output due to aging of the lamp and gradual dirt accumulation on the lamp

What are the primary types of concrete structural systems? (Choose the two that apply.) (A) Beam-and-girder system (B) Cast-in-place (C) One-way system (D) Open-web joist system (E) Precast

(B), (E) The two primary types of concrete structural systems are cast-in-place and precast. At a secondary level there are one-way and two-systems

Which of the following materials is considered a rapidly renewable building material? (A) chestnut flooring sawn from old barn beams (B) mineral fiber insulation (C) linoleum (D) recycled cardboard paneling

(C)

The cost index in a city A is 1257, and the cost index in city B is 1308. The expected construction cost for a building in city A is $1,250,000. What will be the approximate expected cost for the same building if constructed in city B? (A) $1,100,00 (B) $1,200,00 (C) $1,300,00 (D) $1,400,00

(C) $1,300,00 The ratio of expected costs in the two cities is estimated to be equal to the ratio of cost indices. CityA/CityB = IndexA/IndexB

Which medium transmits sound the slowest? (A) Water (B) Steel (C) Air (D) Concrete

(C) Air Sound transmission is related to density. The more dense something is the more easily it transmits sound. Air - 1,139 ft/sec Water - 4,625 ft/sec Wood - 10,825 ft/sec Brick - 11,800 ft/sec Concrete - 12,100 ft/sec Steel - 16,000 ft/sec Glass - 16,400 ft/sec Aluminum - 19,000 ft/sec

A mechanical engineer has proposed two different HVAC systems for a new project. Either system is suitable for the project type and location, but for tax purposes, the owner prefers a higher HVAC equipment depreciation cost over the lifecycle of the system. System A is a forced air heating and cooling system with an upfront cost of $10,000, and anticipated useful life of 20 years, and a salvage value of $1,000. System B is an electric packaged terminal unit with an upfront cost of $7,500, an anticipated useful life of 15 years, and a salvage value of $1,500. What is the annual depreciation of the system with the higher per year depreciation? (A) $300 (B) $400 (C) $450 (D) $500

(C) $450 Annual depreciation value is an important aspect of life cycle considerations. It can be determined by subtracting the salvage value from the initial investment, then dividing by the estimated life span of the system. System A: $10,000 - $1,000 = $9,000 $9,000 / 20 years = $450/year System B: $7,500 - $1,500 = $6,000 $6,000 / 15 years = $400/year

A building has a 240 ft perimeter. 180 ft of the perimeter fronts an open space 24 ft wide. (The building could be a square 60 ft on a side.) Most nearly, the value of the factor 'If' (area factor increase) is (A) 0.1 (B) 0.2 (C) 0.3 (D) 0.4

(C) 0.4 If == area factor increase F == frontage space P == building perimeter W == width of public way or open space (20-30 ft) If = = ( (F/P - .25) * W) / 30 = ( (180/240-.25)*24 / 30 = 0.4

Which material has the lowest perm rating? (A) 10 mil polyethylene (B) gypsum wallboard (C) 1 mil aluminum foil (D) exterior oil paint

(C) 1 mil aluminum foil Vapor retarders are selected based on their perm ratings. The best vapor retarders have the lowest ratings. This problem is essentially asking which of the listed materials is the best vapor retarder, and the answer is aluminum foil with a perm rating of zero. Polyethylene is the next best, with a perm rating of 0.03. Three coats of exterior oil paint have a perm rating of 1.6 to 3.0. Gypsum wallboard allows the most moisture to pass through, with a perm rating of 50

A building includes an office with a gross area of 3,700 ft2 and two training classrooms of 1,200 ft2 each. What is the occupant load for the entire building? (A) 92 occupants (B) 120 occupants (C) 157 occupants (D) 206 occupants

(C) 157 occupants Office occupant load = = (3,700 ft2) / (100 ft2/occupant) = 37 occupants Classrooms = = (2 * 1,200 ft2) / (20 ft2/occupant) = 120 occupants Total = = 37 + 120 = 157 occupants

A contractor is calculating how much lumber he needs to order to build a small addition on a client's house. The three exterior walls are to be framed with 2x6 studs at 16" on center. The room is 12'x12', so he estimates that he will need nine 8' studs for each wall. The contractor will be charged by the board-foot for the lumber he purchases. How many board-feet will he need to buy (A) 108 board-feet (B) 144 board-feet (C) 216 board-feet (D) 243 board-feet

(C) 216 board-feet

A parking lot requires 60 stalls. What is the best estimate for the square footage required for the parking lot? (A) 12,000 ft² (B) 15,000 ft² (C) 24,000 ft² (D) 36,000 ft²

(C) 24,000 ft² Generally speaking, a parking layout will require about 400 ft² per vehicle. For a parking lot with 60 stalls that means: 60*400 ft² = 24,000 ft²

A geared traction elevator would be most appropriate for a(n) (A) 5-story medical office building (B) 16-story office building (C) 4-story department store (D) 8-story apartment building

(C) 4-story department store Geared traction elevators can be designed to serve a wide variety of slower speeds and high capacities, so they are ideal for low-rise buildings with heavy loads, such as department stores A geared traction elevator could be used for a small medical office building, but a higher speed would offer better service A 16-story office building would need a high-speed, moderate capacity elevator, so a geared traction type would be incappropriate An apartment building would require a low capacity but higher speeds

According to model codes, the minimum width of an office exit corridor serving an occupant load of 55 is (A) 36 in (B) 42 in (C) 44 in (D) 48 in

(C) 44 in Model codes prescribe a minimum corridor width of 44 in, with various exceptions. Corridors in residential occupancies or those serving an occupant load of less than 50 may be 36 in wide. Other occupancies require wider corridors, but B occupancies (offices) require the 44 in width Model codes also require that the minimum width of an exit corridor be calculated by multiplying the occupant load by a factor given in the codes, which varies depending on the occupancy, the interior area of the building, and whether the building is sprinklered. Both requirements must be checked, and the larger of the two should be used. If this calculation results in a number larger than other minimum requirements given in the code (such as for corridors), then the larger dimensions must be used

What is typically the maximum height building that can utilize an upfeed water supply system? (A) 20-30 feet (B) 30-40 feet (C) 50-60 feet (D) 70-80 feet

(C) 50-60 feet

A small commercial building with a roof area of 10,000 ft2 is located in an area that receives an annual rainfall of 20 in. 25% of the water is lost in evaporation, run-off, absorption, and impoundment. The amount of rainwater that can be collected in a year is (A) 9,000 gal (B) 15,000 gal (C) 90,000 gal (D) 150,000 gal

(C) 90,000 gal Collection = = (area) (annual rainfall) (1-water loss) (in-to-gal) = (10,000 ft2) (20 in) (1-.25) (.6) = 90,000 gal

Which medium transmits sound the fastest? (A) Concrete (B) Air (C) Aluminum (D) Water

(C) Aluminum Sound transmission is related to density. The more dense something is the more easily it transmits sound. Air - 1,139 ft/sec Water - 4,625 ft/sec Wood - 10,825 ft/sec Brick - 11,800 ft/sec Concrete - 12,100 ft/sec Steel - 16,000 ft/sec Glass - 16,400 ft/sec Aluminum - 19,000 ft/sec

Of the below toilet stall partitions finishes, which has the lowest initial cost? (A) Porcelain enamel (B) Stainless steel (C) Baked enamel (D) Laminated plastic

(C) Baked enamel

A building's occupancy group can be found where? (A) Zoning code (B) Municipal code (C) Building code (D) Fire code

(C) Building code In the IBC, occupancy types are listed and defined in Chapter 3 Use and Occupancy Classification

Which of the following window types allows for the greatest actual openable area? (A) Single-hung (B) Awning (C) Casement (D) Jalousie

(C) Casement Casement allows for - 90% openable area Awning allows for - 75% openable area Jalousie allows for - 75% openable area Single-hung allows for - 45% openable area

An architect is considering general planning concepts for a veterinarian hospital and kennel, both owned by the same company. Which is the best design approach for both? (A) Combine both functions into one structure to allow sharing of common functions (B) Make the hospital two levels to provide more site space for outdoor runs (C) Design two separate buildings that share the same parking area (D) Develop two wings for different functions but share a common entry suite

(C) Design two separate buildings that share the same parking area Although both types of facilities may share some common functions, such as administrative offices and food storage, the services each provides is unique and the reasons for visiting differ. In addition, a kennel requires adjacent outdoor facilities that a hospital does not. A two-story level facility is not efficient for animals or moving supplies because it would require ramps and an elevator. People bring in their animals to each facility for very different reasons, so each should have its own reception area to avoid potential problems The best design approach would be to separate the two buildings but to have them on the same site with separate, distinct entrances. However, they could share the same parking area

Which type of loudspeaker system is most appropriate to install in a low-ceiling area? (A) Direct (B) Indirect (C) Distributed (D) Static (E) Central

(C) Distributed There are only two principal types of loud speaker systems: central and distributed In locations with low ceiling distributed, or overhead, loudspeakers are most appropriate

Which of the following statements about absorption is true? (A) The absorption coefficient of a room should be between 1.20 and 2.50 (B) To produce a clearly noticeable reduction in noise, the absorption in a room should be doubled (C) Even a small hole in a 10'x10' partition, such as that caused by unsealed electrical receptacles placed back to back, can make the partition effectively useless for blocking sound (D) In large rooms, absorptive treatments applied to the walls are more effective than ceiling treatments

(C) Even a small hole in a 10'x10' partition, such as that caused by unsealed electrical receptacles placed back to back, can make the partition effectively useless for blocking sound

Where is wind energy transfer from a shear wall? (A) Building perimeter (B) Diaphragm (C) Foundation (D) Roof

(C) Foundation Lateral loads such as wind are transferred at the foundation of a shear wall

Which of the following types of swinging door features a window? (A) Dutch (B) Flush (C) French (D) Paneled

(C) French A french door is defined by the use of a window in it's design. This is the best answer for this question

Which climate type is most difficult to design for when mechanical cooling is not available? (A) Cold (B) Hot-Arid (C) Hot-Humid (D) Temperate

(C) Hot-Humid A hot-humid climate is most difficult to design for when mechanical cooling is not available because the air must be both humidified and cooled

Which element is found in the greatest proportion in structural steel? (A) Carbon (B) Copper (C) Iron (D) Tin

(C) Iron Steel is mostly comprised of iron. Steel is an alloy of iron and carbon

An architect is developing a site plan for a developed property with an existing building. The building is in good condition and was used for the same purpose as the client's proposed use of the site. Which of the following four strategies would earn the greatest number of points within the Leadership in Energy and Environmental Design (LEED) 2009 for New Construction rating system? (A) Maintain at least 50% of the interior nonstructural elements (B) Remove the building and recycle and/or salvage at least 50% of construction and demolition debris (C) Maintain at least 95% of the existing walls, floors, and roof (D) Reuse materials from the site that amount to 5% by cost, of the total value of materials on the project

(C) Maintain at least 95% of the existing walls, floors, and roof If 95% or more of the existing building's structure, exterior walls, and roof are reused, the LEED 2009 NC rating system awards three points, one point each for reaching levels of 55%, 75%, and 95%. Window assemblies and nonstructural roofing materials are excluded from these calculation

If the calculated deflection of a steel beam too high, it's necessary to select a different beam section with a greater ___________ (A) Area (B) Modulus of elasticity (C) Moment of inertia (D) Section modulus

(C) Moment of inertia The moment of inertia is directly proportional to the amount of deflection. If the deflection is high, the moment of inertia (I) must change Note: This answer is wrong as a section modulus will also reflect the span-to-depth ratio of a beam or wide flange, and the moment of inertia is the numerator in calculating an elastic section modulus The modulus of elasticity is the ratio of stress/strain and gives an idea of a materials resistance to being deformed elastically

What is the best passive cooling strategy during the summer in a hot-humid climate? (A) Design a series of pools and fountains to cool by evaporation (B) Include broad overhangs to shield glass and outdoor activities from the sun (C) Orient the building to catch summer breezes (D) Use light-colored surfaces to reflect sunlight and reduce solar gain

(C) Orient the building to catch summer breezes

Your building is in a climate that experiences hot summers. What would be an effective design strategy to use to promote internal comfort during the summer? (A) Clerestory windows on the west side of the building (B) Skylights (C) Overhangs on the west and south facing windows (D) Louvers on the north side of the building

(C) Overhangs on the west and south facing windows Of the listed strategies, overhangs on the west and south facing windows would be most effective. These overhangs would block direct (heat carrying) sunlight from entering the building during a large portion of the day Clerestory windows and skylights would help bring in light but won't help reduce heat gains. Louvers on the north side of the building could help create privacy but would not help reduce heat gains

What type of agent would be added to the mix when pouring concrete in hot weather? (A) Air-entraining agent (B) Hardening agent (C) Retarding agent (D) Workability agent

(C) Retarding agent Retarding agents delay the setting of concrete

Once soil has been contaminated, the risk of contaminants leaching into groundwater is high. Before this happens, what method of remediation can be used to extract contaminants from unsaturated soils? (A) Penetration radar (B) Phytoremediation (C) Soil vapor extraction (D) Photolysis

(C) Soil vapor extraction Soil vapor extraction is a physical treatment process for in situ remediation of volatile contaminants in unsaturated soils. SVE (also referred to as in situ soil venting or vacuum extraction) is based on mass transfer of contaminant from the solid and liquid phases into the gas phase, with subsequent collection of the gas phase contamination at extraction wells. Extracted contaminant mass in the gas phase is treated in above-ground systems

An architect has been hired to prepare a design for remodeling toilet rooms to make them accessible. The architect finds it is impossible to provide adequate clearance on one side of an entrance door. What is the most economical course of action (A) Apply to the building department for a hardship exemption because compliance is not readily achievable (B) Tell the client that walls should be demolished and the toilet rooms replanned to provide the necessary clearances (C) Specify a power-assisted door opener that meets accessibility standards for the noncompliant door (D) Plan for accessible toilet rooms in other location in the building where all requirements can be adequately met

(C) Specify a power-assisted door opener that meets accessibility standards for the noncompliant door A power-assisted door would be the least expensive option and require the least construction time It would be possible to solve the problem by demolishing the existing restrooms and rebuilding them to comply with accessibility standards, or meet the requirement for accessible facilities in another location in the building, but both of these options are likely to be less economical than the door opener

During preliminary planning for a middle school, the architect realizes that the school district facilities management department requires a larger building than is allowed by current zoning setbacks. How can the architect resolve the conflict? (A) Ask the school board to reduce the space needs (B) Appeal to the planning commision (C) Suggest the school apply for a variance (D) Apply for a conditional use permit

(C) Suggest the school apply for a variance The school board is unlikely to reduce space needs The local planning commission is not involved with setback issues A conditional use permit typically is used to allow a nonconforming use in a zoning district A variance is a deviation from zoning regulations

A historic colonial house has exterior walls constructed of stone with wood framed floors and roof. What construction type is this building? (A) Type I (B) Type II (C) Type III (D) Type IV (E) Type V

(C) Type III Type I is made entirely of noncombustible materials such as concrete or stone Type II is made of noncombustible materials with the exception of the roof covering material Type III has masonry-bearing walls but floors, structural framework, and roof made of wood or other combustible materials Type IV is heavy timber Type V is construction which is entirely combustible wood framed construction

What is considered the ideal range of temperature and humidity to establish human comfort? (A) Winter: 50-61°F Summer: 80-85°F Relative Humidity: 40-80% (B) Winter: 55-65°F Summer: 80-90°F Relative Humidity: 55-95% (C) Winter: 62-72° Summer: 65-75° Relative Humidity: 30-60% (D) Winter: 60-70°F Summer: 70-80°F Relative Humidity: 20-50%

(C) Winter: 62-72° Summer: 65-75° Relative Humidity: 30-60% While thermal comfort has many different factors that can affect an occupant's perceived comfort level, it is generally accepted that the comfortable ranges are: Winter: 62-72° Summer: 65-75° Relative Humidity: 30-60%

What is not true about zoning? (A) The first zoning ordinance was introduced in 1916 in New York (B) Zoning is the most common legal constraint on land development (C) Zoning has had legal status for centuries (D) Zoning was originally introduced to alleviate some of the issues associated with population booms in cities

(C) Zoning has had legal status for centuries Zoning wasn't introduced in the US until 1908 in Los Angeles and 1916 in New York City

An architect is designing an exterior pedestrian walk. In one area, there is a change in level of 12". The transition must be made within a horizontal distance of no more than 14'. Which of the following design solutions is most appropriate? (A) two steps with 6" risers (B) three steps with 4" risers (C) a ramp 14 ft long (D) two steps with 6" risers and a ramp 12' long

(C) a ramp 14 ft long

What is the optimal location for an elevator machine room? (A) beside the elevator on the lowest level of the building (B) adjacent to the electrical room (C) above the hoistway (D) adjacent to the mechanical room

(C) above the hoistway Ideally, elevator mechanical rooms should be placed above the hoistway. They must be at least as wide as the elevator shaft and generally need to be more than 12' deeper than the hoistway, but the exact size of the space should be calculated considering the type of elevator specified, the sizes of all the equipment to be housed in the space, and the required servicing clearances

Where does a common path of egress travel end? (A) in an area of refuge (B) at the exterior exit from the building (C) at the point where an individual has a choice about which direction to go to reach an exit (D) at a public way

(C) at the point where an individual has a choice about which direction to go to reach an exit

Which type of heating, ventilating, and air conditioning (HVAC) system would be the best choice for a large building where a need for simultaneous heating and cooling is expected? (A) direct expansion (B) variable air volume (C) dual duct (D) reheat system

(C) dual duct Hot and cold air are both constantly available in a dual duct system; it can provide heating and cooling simultaneously (a multi-zone system also has this capability). However, the dual duct system has some drawbacks; it is relatively expensive, requires a lot of ductwork (two runs to each space), and is not very energy efficient. Direct expansion, variable air volume, and reheat systems provide heating and cooling but cannot do both at the same time

An architect is designing a new furniture gallery to be housed within an old bank building. The store will feature modern furniture and artwork and will display approximately one million dollars' worth of inventory. Which of these types of sprinkler systems will be the most appropriate choice for the store? (A) wet pipe (B) dry pipe (C) preaction (D) deluge

(C) preaction

Potential overheating of a medical clinic in a temperate climate could be minimized by (A) designing an overhang for the west and east sides of the building (B) planning a building shape to minimize the surface area of south-facing walls (C) having a landscape architect specify deciduous trees near the south elevation (D) using reflective glass on the west side of the building

(C) having a landscape architect specify deciduous trees near the south elevation Option (A) is incorrect because overhangs are not effective on the west and east sides of a building due to the low sun angle. Vertical louvers, or fins, are more effective in these locations Option (B) is incorrect because the south side actually receives less solar radiation than the east or west sides because the sun is high during the middle of the day. It would be more effective to minimize the roof area to cut down on solar radiation Option (D) is incorrect because although reflective glass would reduce heat gain, it reflects sunlight on neighboring buildings and the landscape

The walls of a new coffee shop will be painted a rich pumpkin orange. What type of lighting will provide the best overall color rendering and accent the orange walls? (A) cool white fluorescent (B) warm white fluorescent (C) incandescent (D) daylight

(C) incandescent

In order to achieve the most uniform, straight-grain appearance in wood paneling, which of the following should be specified? (A) plain slicing (B) rotary slicing (C) quarter slicing (D) half-round slicing

(C) quarter slicing Plain slicing produces a figured pattern with a characteristic "cathedral" appearance Rotary slicing produces the most varied grain pattern, and half-round slicing yields a moderate amount of pattern Because quarter slicing cuts perpendicular to the growth rings, this gives the straightest pattern of the choices listed. Rift slicing would also give a very uniform grain pattern

Which building type is most appropriate for a retail store in an urban setting? (A) courtyard building (B) perimeter yard building (C) rear yard building (D) side yard building

(C) rear yard building

The straightest, most uniform grain appearance in board lumber is achieved by specifying (A) plain sawing (B) quarter sawing (C) rift sawing (D) rotary sawing

(C) rift sawing With the rift sawing method, each cut for a board is made by sawing a quarter section of log radially toward the center point of the tree. This requires the quarter section of log being sawn to be shifted slightly for each cut. The grain in the resulting boards is nearly perpendicular to the face of the board. This gives the straightest grain pattern. Rift sawing is normally reserved for oak, to reduce the appearance of flaking, which is caused by medullary cells in the oak Quarter sawing is similar to rift sawing except that the quartered log is held in a stationary position as the cuts are made toward the center point of the tree. Yields for this type of sawing are higher than those for rift sawing, but boards made by cutting away from the center will have grains at a slight angle to the face of the board Plain sawing cuts an entire log in one direction. Although plain sawing makes the most efficient use of the tree, boards cut near the tree edges will have a less uniform grain pattern Rotary sawing is not an accurate term; rotary slicing is used only for veneer, not boards

In order to minimize the space required for wood floor framing, the architect would most likely detail the connections to show the use of (A) hurricane straps (B) post caps (C) saddle hangers (D) splice plates

(C) saddle hangers Saddle hangers are pieces of preformed metal, designed to fit over a beam, that provide supports for joists framed perpendicularly to the beam. This type of connection hardware allows the joists to be installed with their top edges flush with the top edges of the beams. It avoids the requirement that the joists be placed over the beams, which would increase the total depth required for the floor structure

In order to get a countertop or cabinet to fit snugly against a slightly irregular partition, which of the following should be specified or called out on the drawings? (A) astragal (B) extended frame (C) scribe (D) shoe molding

(C) scribe A scribe is an integral part of the woodwork or a separate piece of trim that is cut, sanded, or otherwise shaped on the job site to exactly match the irregularities of an adjacent material

Which of the following is an ideal design strategy for a building in a hot-arid climate? (A) minimizing thermal mass (B) locating large windows for cross ventilation (C) shading east and west facing windows (D) using a compact form

(C) shading east and west facing windows

A seven-story office building is to have a variable air volume system. The building will have 105,000 ft2 of net space and an estimated 126,000 ft2 of gross area. About how much space should be the minimum area allowed for heating, ventilating, and air conditioning (HVAC) systems? (A) 2,500 ft2 (B) 3,800 ft2 (C) 6,300 ft2 (D) 7,600 ft2

(D) 7,600 ft2 For most midsize buildings, an all-air or air-water system needs about 3-9% of the gross area for HVAC system mechanical space. Office buildings fall somewhere near the midpoint of the range, so use 6% for this question.

In the design of a manufacturing plant and its associated office space, the primary form determinant is most likely (A) construction schedule (B) mechanical systems (C) structure (D) sustainability

(C) structure When considering just form, the most likely parameter driving a manufacturing plant's building configuration is structure due to the need to have large, open spaces. This requirement would translate into a design of a simple post-and-beam system of widely spaced columns or a long-span structure, such as an arch or space frame The other options are secondary influences because scheduling, mechanical systems, and sustainability measures easily can be integrated once the basic structure is determined. Even visible sustainability features, such as natural lighting or solar panels, would have to be designed in response to the size and shape of the manufacturing area

In a life-cycle cost analysis, costs incurred prior to the baseline date are referred to as (A) uniform costs (B) break-even costs (C) sunk costs (D) sensitive costs

(C) sunk costs Sunk costs are expenses that have been incurred on a project prior to the baseline date and cannot be recovered. Generally these costs are disregarded when analyzing life-cycle costs, because it is impossible to make decisions about the best way to spend money that has already been spent. Sunk costs are incorporated into analyses of total project costs

A client who has recently moved from New York to southern California has hired an architect to design a new house in the colonial revival style, which is similar to a former home. The architect should recognize that this building configuration request may be influenced by (A) cultural attitudes (B) status seeking (C) symbolism (D) regionalism

(C) symbolism The client's cultural upbringing is not known, so cultural attitudes cannot fully account for the client's style preference. The client, as well as the architect, would probably see the cultural setting of California as very different from New York Building a house, even a very large mansion probably would not convey an image of status in southern California. It is more likely that if the client wanted status, the architect should design a large, unique house in a contemporary style Regionalism is not a factor because the architectural styles of contemporary southern California are typically quite different from the northeast US Such a request is an example of how a certain style of building symbolizes what a home should look like, so the factor most likely driving the client's request is symbolism

When specifying a hardwood floor over a concrete slab on grade, which of the following should the architect also specify? (A) 3/4" plywood subflooring and 15# building felt (B) 3/4" tongue-and-groove plywood placed over a layer of mastic (C) treated wood sleepers on mastic and a layer of polyethylene vapor barrier (D) 15# building felt

(C) treated wood sleepers on mastic and a layer of polyethylene vapor barrier Sleepers are generally 2x4s laid flat at 16" on-center. A layer of polyethylene vapor barrier should be placed over the sleepers but under the finish flooring material

Which of the following is the most commonly used method to disinfect water to make it potable? (A) chlorination (B) ozonation (C) ultraviolet light (D) zeolite process

(C) ultraviolet light

Wood I-joists generally can be used for spans (A) up to about 20 ft (B) up to about 30 ft (C) up to about 45 ft (D) exceeding 100 ft

(C) up to about 45 ft

An architect is designing a nursing home with 150 beds. The patients have varying levels of mobility and independence. The building manager requests a HVAC system that permits each patient to control the temperature in his or her own room, that is quiet, and that requires minimal maintenance. Which type of system will be the most appropriate recommendation? (A) packaged terminal units (B) fan coil terminals (C) variable air volume (D) single duct, constant air volume

(C) variable air volume a variable air volume (VAV) system will be the best choice for this application. VAV systems allow for individual control of temperature, quiet operation, and minimal maintenance Packaged terminal units and fan coil terminals permit control over the temperatures of individual spaces but do not operate as quietly or require as little maintenance as VAV systems Single duct, constant air volume (CVAV) units are relatively inexpensive to install and maintain, but do not offer occupants the ability to control the temperatures of individual spaces

Why is steel used to reinforce concrete? (A) Improve bond strength (B) Improve durability (C) Improve rigidity (D) Improve tensile strength (E) Improve compressive strength

(D) Improve tensile strength Steel is used to reinforce concrete because concrete is strong is compression but not tension. Including steel rebar in concrete improves the concrete's strength in tension. Both have similar thermal rates of expansion

A soils report has indicated that the water table is 5'-0" above the basement level of a planned three-story building. What type of construction technique most likely will be required? (A) dampproofing (B) surcharging (C) waterproofing (D) waterstopping

(C) waterproofing Waterproofing is the control of water and moisture that is subject to hydrostatic pressure. It can refer to the application of watertight membranes, waterstops, of bentonite panels when building below the water table Dampproofing is the control of water and moisture when hydrostatic pressure is not present Surcharging is the preloading of the ground with fill material to cause consolidation and settlement of the underlying soil. Surcharging is used to increase the bearing capacity of soil or to decrease possible settlement, or both A waterstop is a preformed piece of material used to seal construction joints. Although waterstops would be used in this situation, they are a subset of the larger requirement to waterproof the entire basement slab and a portion of the basement walls

Your client has just purchased a new home they'd like to renovate. The client expresses concern about the slope of the site (2:12) being too steep and eroding. What are the best actions to take? (Choose the two that apply.) (A) Regrade the site to 1:12 (B) Construct a planter wall at the base of the slope (C) Recommend planting vegetation (D) Inform the client that erosion isn't a concern (E) Split the slope in half with a retaining wall to create two flat areas

(C), (D) Although the client may have expressed concern about erosion, the slope of the site is not a concern for erosion until around 25%. The slope on this site is 2:12 or 16.67% The best options for the architect to take would be to inform the client that erosion isn't a concern and recommend planting vegetation if that might make the client feel more at ease Regrading the site, building a planter wall, and building a retaining wall are expensive alternatives that aren't necessary for this condition

Which are the most-common products where asbestos might be present in existing construction? (Choose all that apply.) (A) Planter Pots (B) Window frames (C) Tiles (D) Insulation (E) Siding

(C), (D), (E) Asbestos fibers virtually indestructible which lead to it's widespread use in the 50s and 60s. Asbestos fibers were often combined with other materials for use in thousands of industrial, maritime, automotive, scientific and building products. The following list shows the range of materials that could contain asbestos: Insulation Fireproofing/Acoustical Texture Products Textile and Cloth Products Spackling, Patching & Taping Compounds Gaskets and Packings Asbestos-cement Pipe and Sheet Material Tiles, Wallboard, Siding and Roofing Friction Materials Vermiculite Laboratory hoods and table tops

Which of the following would an architect be most concerned about when designing the lighting for an office space with computer workstations and standard desks? (Choose the three that apply.) (A) color rendering index (B) visual comfort probability (C) task/surrounds brightness ratio (D) veiling reflections (E) reflected glare (F) lighting uniformity

(C), (D), (E) In an office space where computer monitors and standard office tasks are present, the architect should be concerned with two results of glare. Veiling reflection would be of concern for standard office tasks such as writing and reading whereas reflected glare would be critical in using monitors. The brightness ratios between the tasks and their surroundings are important, especially when employees spend hours in front of a computer monitor The color rendering index, visual comfort probability, and lighting uniformity are less important

Which of the following are materials most commonly used to make rigid frames? (Choose the three that apply.) (A) Brick masonry (B) Glass block (C) Reinforced concrete (D) Structural steel (E) Wood

(C), (D), (E) Rigid frames are most commonly made from reinforced concrete, steel, and wood. If the columns and beams in the frame are able to resist both forces and moments, the assembly becomes a rigid frame

Which of the following is laminated glass commonly used for? (Choose the four that apply.) (A) acoustical control (B) decorative purposes (C) fire protection (D) insulation (E) safety (F) security

(C), (D), (E), (F)

How can the health risks of asbestos found in building renovation be avoided? (Choose the two that apply.) (A) Heat-drying (B) Adequate cross-ventilation (C) Abatement (D) Vaccuming (E) Encapsulation

(C), (E) If asbestos is discovered during construction a specialist should be hired to assess the situation. The specialist will most likely recommend abatement (removing) the asbestos or encapsulating (sealing) it. Asbestos is only problematic when it becomes airborne

Which of the following structures require some complex wind calculations and possibly some wind tunnel testing? (Choose the three that apply.) (A) commercial building with a height of 50 ft (B) residential building with a height of 200 ft (C) office building with a height of 450 ft (D) building with a height-to-width ratio of 3 (E) building with a height-to-width ratio of 7 (F) extra-long span suspension bridge

(C), (E), (F) Generally, complex wind calculations and wind tunnel testing are required for buildings with heights exceeding 400ft, for buildings subject to dynamic effects, those sensitive to wind vibrations, and for buildings with a height-to-width ratio of 5 or more. Wind tunnel testing is also often carried out on reduced-scale models of long-span suspension bridges

Which of the following statements is true? (A) Damp-proofing controls moisture that is under hydrostatic pressure (B) Membrane coatings should always be used for damp-proofing (C) Waterproofing membranes may be easily punctured and require a protective covering (D) If hydrostatic pressure is present, foundation drains will be of no use

(D)

How much space should be provided between shared plumbing walls with wall mounted toilets? (A) 12" (B) 16" (C) 18" (D) 24"

(D) 24" The stud cavity should be 24" (space provided between the studs). Typically provide 12" behind the stud for a wall mounted carrier. If the toilets were not wall mounted, 16" would be sufficient

Which of the following is not a standard nominal width for an escalator? (A) 32 in (B) 40 in (C) 48 in (D) 54 in

(D) 54 in Most manufacturers make escalators in nominal widths of 32", 40", and 48" with corresponding actual widths of 24", 32", and 40"

In their bid, the contractor has included a line item for escalation. What is escalation? (A) Initial costs to get a project started (B) A portion of money set aside for any legal fees (C) A contingency (D) Increase in price due to anticipated inflation

(D) Increase in price due to anticipated inflation Increase in price, specially due to inflation. Escalation clauses are typically included on large construction projects, where the job might exceed one year

Which of the following statements is true? (A) Spalling occurs when water-soluble salts in masonry units or mortar leach out of the brick (B) Tuck pointing is used to finish mortar joints during construction of a new brick wall (C) Flashing in a masonry wall should be terminated just before the face of the brick for best appearance (D) A concealed flashing in a masonry wall with a concrete backup should terminate in a reglet

(D) A concealed flashing in a masonry wall with a concrete backup should terminate in a reglet Reglets are horizontal grooves cast into concrete that allow a piece of flashing to be slipped inside and then carried across the airspace and through the brick for proper drainage and moisture control Efflorescence (not spalling) occurs when salts leach out of a masonry assembly. This produces a white powdery substance that stains the face of the brick Tuck pointing is a process used to repair failing mortar joints. It involves removing the deteriorated mortar to a certain depth and inserting new, compatible mortar into the space, then striking it with a new, water-resistant edge treatment Flashing should always be extended at least 3/4" beyond the face of the brick and turn down at a 45 degree angle for proper drainage. If the flashing is terminated before the face of the brick, the moisture will seep into the brick and mortar and can cause damage. The 45 degree bend provides a drip edge that leads the moisture away from the face of the brick

A building's construction type would not affect the design of which one the below items? (A) Building materials (B) Floor area (C) Building height (D) Accessibility requirements

(D) Accessibility requirements Floor area, building height, and building materials are all affected by a building's construction type. Table 503 in the IBC shows the relationship between these three Accessibility requirements are not affected by construction type

What is the primary factor that determines comfort? (A) Air speed (B) Humidity (C) Perceived user control (D) Air temperature

(D) Air temperature The primary factor of user comfort is air temperature. A 100°F day is going to feel hot regardless of other conditions. A general comfortable air temperature ranges between 69°F and 80°F, though these vary based on other factors

Which type of wood heating device is most efficient at heating a space? (A) Somewhat airtight stove (B) Nonairtight stove (C) Fireplace (D) Airtight stove

(D) Airtight stove

Trusses are used ___________ (A) Diagonally (B) Horizontally (C) Vertically (D) All of the above

(D) All of the above Trusses are used horizontally, vertically, and diagonally. The triangular arrangements of truss components make them suitable in all three conditions

What material contains the most embodied energy? (A) Steel (B) Douglas fir (C) Granite (D) Aluminum (E) Bamboo

(D) Aluminum Of all the standard building materials, aluminum has the most embodied energy (the sum of energy used to create it)

In a project you decide to use a passive cooling strategy of stack ventilation. What is important to maintain? (A) A warm attic or loft space (B) An inside temperature warmer than the inside temperature (C) A warm basement temperature (D) An outside temperature cooler than the inside temperature

(D) An outside temperature cooler than the inside temperature Stack ventilation only works when the outside temperature is cooler than the inside In the stack effect warm inside air is pushed up and out of the building by air entering from the outside low points. The cooler outdoor air pushes the hot air up into the stack

Which of the following programmatic elements would require the most electric load per square foot for an electric air conditioning system? (A) Medical center (B) Office building (C) Hotel room (D) Auditorium seating

(D) Auditorium seating In Auditorium seating, medical center, and office building we're concerned solely with heat generated by a quantity of people doing very light activity. Since the auditorium seating has many more people, the estimated load will be higher Estimated electric loads per SF Auditorium seating: 10-20 Volt-Amperes Restaurant: 6-10 Volt-Amperes Medical Center: 4-7 Volt-Amperes Office building: 4-7 Volt-Amperes

A 4-story building is located in a highly seismic zone and the engineer recommends a structural system that allows the superstructure and substructure to move somewhat independently from each other. What is this system called? (A) Lateral rolling (B) Underpinning (C) Tuned mass damping (D) Base isolation

(D) Base isolation Base isolators allow the superstructure and substructure to move somewhat independently when lateral forces are applied

What are conifers best used for in site design? (A) Providing shade (B) Adding lush greenery to the landscape (C) Separating site uses (D) Blocking wind and views

(D) Blocking wind and views Since conifers maintain their needles all year long, they are best used in site design to block wind and views. They provide shade, but do so year long, even in the winter months when shade isn't desired To help remember the difference between conifers and deciduous trees, it may help to know that in Latin conifer means cone-bearing (think pine cones)

During the design of a restaurant, the architect discovers that a local building code amendment conflicts with state health department requirements. What action should the architect take? (A) Appeal to the health department (B) Ask for a ruling from the building department (C) Have the client resolve the issue (D) Design to the most restrictive requirement

(D) Design to the most restrictive requirement

While developing a site design, what would be the most effective use of deciduous trees? (A) For aesthetic enhancement (B) To block wind and views (C) To separate the site into different areas or zones (D) For shade

(D) For shade Deciduous trees are great for shade because they block the harsh summer sun with their leaves. Conversely, since deciduous trees loose their leaves in the winter, sunlight is able to pass through leafless branches and onto buildings to warm them up Coniferous trees are good for blocking wind and views

While developing a site design, what would be the most effective use of deciduous trees? (A) For aesthetic enhancement (B) To block wind and views (C) To separate the site into different areas or zones (D) For shade

(D) For shade Deciduous trees are great for shade because they block the harsh summer sun with their leaves. Conversely, since deciduous trees loose their leaves in the winter, sunlight is able to pass through leafless branches and onto buildings to warm them up Coniferous trees are good for blocking wind and views

Many older buildings contain lead paint. Why is it considered a hazardous material? (A) Lead paint is not a hazardous material (B) Lead paint in bathrooms can off-gas and enter the water supply (C) It can cause joint aches if ingested (D) It can cause neurological disorders and birth defects

(D) It can cause neurological disorders and birth defects Lead interferes with a variety of body processes and is toxic to many organs and tissues including the heart, bones, intestines, kidneys, and reproductive and nervous systems. It interferes with the development of the nervous system and is therefore particularly toxic to children, causing potentially permanent learning and behavior disorders

The elements which comprise an exit to the outside, including aisles, corridors, doors, and stairways are called what? (A) Exit discharge (B) Egress path (C) Exit (D) Means of egress (E) Right of way

(D) Means of egress The correct answer is means of egress. As defined in the IBC, means of egress is: "A continuous and unobstructed path of vertical and horizontal egress travel from any occupied portion of a building or structure to a public way. A means of egress consists of three separate and distinct parts: the exit access, the exit and the exit discharge."

Which of the following statements is incorrect? (A) A 1-1/2% slope is suitable for rough paving (B) Landscaped areas near buildings should have at least a 2% slope away from the structure (C) A safe sidewalk slopes no more than 2-1/2% (D) Roads in northern climates can safely have up to a 12% grade

(D) Roads in northern climates can safely have up to a 12% grade Most roads should be kept at a grade of less than 10%; very short roads and parking garage ramps are exceptions. In northern climates, where snow and ice are a problem, it is even more important to maintain gentle slopes. A 12% grade would not be safe and could make driving difficult

What type of brick would most likely be specified for an eastern exposure in New Hampshire? (A) NW (B) FBX (C) MW (D) SW

(D) SW SW stands for severe weathering and would be the type that should be specified for the northeastern US NW is normal weathering MW is moderate weathering FBX refers to the finish appearance

Which of the following are examples of a fine aggregate? (Choose all that apply.) (A) Crushed rock (B) Gravel (C) Pebbles (D) Sand (E) Bedrock

(D) Sand Sand is the only listed item that is considered a fine aggregate. Fine aggregates have particles up to a minimum size of 0.075 mm and are tested via a sieve stack

Which of the following is not an ingredient in portland cement? (A) Alumina (B) Iron oxide (C) Limestone (D) Silicon

(D) Silicon To create portland cement, materials that contain appropriate amounts of calcium compounds, silica, alumina and iron oxide are crushed and screened and placed in a rotating cement kiln. Ingredients used in this process are typically materials such as limestone, marl, shale, iron ore, clay, and fly ash Silica is a common oxide form of silicon. Silicon is commercially prepared using silica in an electric arc furnace. Both silica and silicon have lattice structures. But silica differs from silicon due to the presence of silicon-oxygen covalent bonding. This accounts for all the different properties between two

Which of the following statements regarding blast-resistant design is true? (A) L-shaped buildings can minimize blast effects (B) Float glass can be used if properly framed (C) Blast energy decreases in inverse proportion to the square of the distance (D) Standoff distance should be maximized

(D) Standoff distance should be maximized Blast distance decreases exponentially with increased distance between the source of the blast and the building. Because of this, one of the most effective site planning strategies is to maximize the distance between the building and the outermost secured perimeter

What is the most common climate type in the United States? (A) Cold (B) Hot Arid (C) Hot Humid (D) Temperate

(D) Temperate

What is the best definition of occupant load? (A) The estimated amount of persons assumed to occupy a space at any given time (B) The maximum number of persons permitted in a building at any given time (C) The average number of persons permitted in a building at any given time (D) The number of persons for which the means of egress of a building or portion thereof is designed

(D) The number of persons for which the means of egress of a building or portion thereof is designed According to the 2012 IBC: "Occupant load is the number of persons for which the means of egress of a building or portion thereof is designed"

At what span dimension is a structure considered to be long span? (A) 30 feet (B) 40 feet (C) 50 feet (D) There is no set distance

(D) There is no set distance

Why would high-pressure sodium lamps be favored over low-pressure sodium lamps in a storage warehouse? (A) They are less expensive (B) They have a longer lamp life (C) They can operate at higher, more efficient voltages (D) They have better color-rendering properties

(D) They have better color-rendering properties Low pressure sodium lamps produce a monochromatic yellow light that would not be appropriate in a storage warehouse where people may have to discriminate between colors

Which type of portland cement is used in concrete exposed to severe sulfate action? (A) Type 1 (B) Type I (C) Type III (D) Type V (E) Type A

(D) Type V Type I --- General purpose + General construction (most buildings, bridges, pavements, precast units, etc) Type II -- Moderate sulfate resistance + Structures exposed to soil or water containing sulfate ions Type III - High early strength + Rapid construction, cold weather concreting Type IV - Low heat of hydration (slow reacting) + Massive structures such as dams (now rare) Type V -- High sulfate resistance + Structures exposed to high levels of sulfate ions

Which of the following factors accounts for the highest cost of a lighting system over time? (A) lamps (B) luminaires (C) installation (D) operation

(D) operation Over the life-cycle of a lighting system, the continuing operating cost for electricity is the single largest expense For nonresidential applications, maintenance (replacement of lamps and cleaning luminaires, etc) is the second greatest expense

In some cases, a planning department may allow a building to deviate from the zoning code. What is this called? (A) Conditional use permit (B) Non-conforming use permit (C) A "grandfather clause" (D) Variance

(D) Variance In the event that a building deviates from the zoning code, the design team can apply for a variance from the local jurisdiction. If granted, a variance gives permission for a design to deviate from the zoning code Nonconforming uses are for older buildings that were constructed before the current code and are out of compliance but allowed to be so as long as they remain (substantially) unaltered A conditional use is similar to a variance but only refers to deviations of use (not the zoning code)

Which of the following is not a major type of wood truss? (A) Bowstring (B) Parallel chord (C) Triangular (D) Webbed chord

(D) Webbed chord Webbed chord trusses are a fictional type of truss and therefore not a major type of wood truss

When would underpinning be required during construction? (A) When soils starts to heave up a slab on grade (B) When the finish floor starts to separate from the subfloor (C) When a structural beam starts to rotate more than expected (D) When the original foundation is not strong or stable enough

(D) When the original foundation is not strong or stable enough In construction or renovation, underpinning is the process of strengthening the foundation of an existing building or other structure. Underpinning may be necessary for a variety of reasons: + The original foundation is simply not strong or stable enough + The usage of the structure has changed + The properties of the soil supporting the foundation may have changed (possibly through subsidence) or were mischaracterized during design + The construction of nearby structures necessitates the excavation of soil supporting existing foundations + To increase the depth or load capacity of existing foundations to support the addition of another story to the building (above or below grade) + It is more economical, due to land price or otherwise, to work on the present structure's foundation than to build a new one + Earthquake, flood, drought or other natural causes have caused the structure to move, thereby requiring stabilization of foundation soils and/or footings

Which material cannot be utilized as a structural component in noncombustible construction? (A) Concrete (B) Masonry (C) Steel (D) Wood

(D) Wood

When is the ideal time to "value engineer" a project? (A) in the initial planning stages (B) at the end of the design development phase (C) after the construction documents have been completed (D) after bids are received and they all exceed the owner's budget

(D) after bids are received and they all exceed the owner's budget

Wastewater flows because of differences in elevation (A) between catch basin entrances (B) between storm sewer vents (C) between drain inlets (D) along inverts

(D) along inverts The difference in elevation between two points are the bottom, or invert, of a sewer line is what causes the water to flow. The term invert is also used to call out the bottom of drains, catch basins, and manholes

For use in designing buildings and structures for wind forces, the ASCE/SE17 standard defines three different wind exposure categories. In which category is the wind most severe? (A) category A (B) category B (C) category C (D) category D

(D) category D The three wind exposure categories are labeled B, C, and D Category B refers to urban and suburban wooded areas and terrain with obstructions Category C refers to open terrain, such as in a desert area The most severe wind exposure is in category D, which refers to flat and unobstructed terrain near large bodies of water

A power distribution system is to be specified for an open office area. Which of the following is the most flexible and economical alternative (A) access floor system (B) underfloor raceway system (C) cellular metal floor raceways (D) ceiling raceway system with pole raceways

(D) ceiling raceway system with pole raceways Any of the options would allow flexibility, but the ceiling raceway system with pole raceways would be the least expensive An access floor system consists of individual panels, typically 24" square, supported on adjustable pedestals above the structural floor. These systems provide unlimited flexibility for routing power, communications, and air supply, but are expensive and not justified unless a great deal of cabling is involved or future changes will be extensive (server room) An underfloor raceway system consists of a series of parallel rectangular metal raceways laid on the structural slab and covered with concrete A cellular metal floor raceway is similar in concept, but is part of the structure

Which of the following outdoor deck materials is most comfortable and best moderates the microclimate around a house in a temperate zone? (A) red brick pavers (B) light colored concrete (C) grass between stone pavers (D) dark wood decking

(D) dark wood decking

Which heating, ventilating, and air conditioning (HVAC) system would be the most appropriate choice for a hospital? (A) multizone (B) high-velocity dual duct (C) variable air volume (D) fan coil with supplementary air

(D) fan coil with supplementary air A system that exhausts all return air would be the best choice to maintain the quantities of fresh air needed in a hospital. A fan coil with supplementary air would satisfy this requirement All of the other systems listed return room air to the main air handling unit, where some of it is reused in the system

Which of the following types of elevators is used for high-rise office buildings? (A) standard hydraulic (B) high-speed hydraulic (C) geared traction (D) gearless traction

(D) gearless traction Gearless traction is one type of mechanism used for high-speed elevators, which a high-rise building would require Geared traction elevators are used for low speed and high capacity Hydraulic elevators are not appropriate for high-rise buildings; they are used for buildings from two to five stories, or up to about 50 ft

A soil investigation for a building site reveals that the soil type is sandy clay and that bentonite is present. Which of the following foundation types would be most appropriate? (A) spread footings (B) mat foundation (C) belled piers (D) grade beam on piers

(D) grade beam on piers Grade beams on piers are used where expansive soil such as bentonite is present. The beams transfer the building weight to the piers, which are commonly placed on bedrock. Voids under the beams allow the soil to expand without heaving the foundation Each soil type has a certain bearing capacity, which is the load (measured in pounds per square foot or kilopascals) from a building foundation that the soil can resist. Of the various soil types, bedrock and sedimentary rock have the highest bearing capacities

On a site with extensive development of buildings, roads, and parking, which change in drainage would have the most significant negative impact? (A) decrease in pervious paving (B) increase in water held on site (C) increase in the number of drainage grates (D) increase in the overall runoff coefficient

(D) increase in the overall runoff coefficient The runoff coefficient is the fraction of the total precipitation falling on a surface that runs off the surface or is not absorbed into the ground. Although the runoff coefficients may vary slightly depending on the types of paving materials used, on a site with a great deal of hard-surface development, more stormwater would have to be diverted to a storm drainage system or to natural waterways. It is preferable to minimize the amount of water allowed to run off a site The runoff coefficient is used to calculate the amount of runoff in cubic feet per second on a site. The calculation takes into account the rainfall intensity and the area of the site. The runoff coefficient value ranges from almost 0 for wooded areas with spongy soil to 1.0 for totally waterproof surfaces. The formula is Q=CIA where C is the runoff coefficient, I is the rainfall intensity in inches per hour, and A is the area of the surface in acres

The height of a proposed building will be most influenced by the decision to use (A) daylighting (B) indirect lighting (C) underfloor ventilation systems (D) interstitial spaces

(D) interstitial spaces

Fire-cut joists are required in (A) platform framing (B) heavy timber framing (C) concrete walls (D) masonry walls

(D) masonry walls A fire-cut joist is one with the ends cut at an angle such that the longer end rests on a masonry bearing wall and the shorter end is flush with the inside face of the wall Fire-cut joists are required in masonry walls to prevent the masonry from being pushed up and out if the wood member should collapse during a fire

Which of the following is the most important consideration in detailing a wood-strip floor? (A) flame-spread rating (B) expansion space at the perimeter (C) nailing method (D) moisture protection from below

(D) moisture protection from below All of the choices listed are considerations in detailing wood floors, so select the most important. Moisture is one of the biggest problems with wood floors, and keeping moisture out in the first place would minimize other problems such as expansion at the perimeter

An architect is designing a four-story hotel. The architect wishes to expose the structural system while maintaining a high fire-resistance rating and needs to minimize construction time. Which type of framing system should the architect choose? (A) wood platform (B) light-gage steel framing (C) steel frame with rigid connections (D) one-way solid slab concrete frame

(D) one-way solid slab concrete frame Cast-in-place concrete one-way solid slab systems allow the designer to expose the structural system while maintaining a high fire-resistance rating. Construction time can be minimized with this system by designing repetitive elements so formwork can be reused. The system can be used with either bearing walls (less expensive, good for repetitive elements) or beams and girders (more expensive, but more flexible for longer spans or greater loads) Neither wood platform framing nor either type of steel framing permits the structure to be exposed and maintain required fire ratings While the steel frame with rigid connections my help with construction time, neither the light-gage steel framing system, nor the wood platform frame helps to speed up construction of a four-story building

A building is being planned for the storage of industrial equipment and machinery. What is the most appropriate reinforced concrete slab system for this building? (A) flat slab on columns (B) flat slab on columns equipped with shearheads (C) flat slab on beams on all four sides (D) pan joist concrete deck system

(D) pan joist concrete deck system The most appropriate reinforced concrete slab system for this building is a pan joist system. It is the strongest system of the options listed, and it is the best suited for the heavy live loads of industrial and storage buildings A flat slab on columns is generally used for light loads such as in residential buildings. Adding shearheads or beams on all four side of the flat slab resolves the punching shear problem around the columns, and allows the system to be used for slightly longer spans and heavier loads, but would still be insufficient for storing industrial equipment and machinery

Which of the following materials provides the highest insulation value (R-value)? (A) expanded perlite (B) expanded polystyrene (C) fiberglass (D) polyisocyanurate

(D) polyisocyanurate Polyisocyanurate has the highest R-value. For 1" thickness, its R-value ranges from 6.25 - 7.2 ft2-hr-F/Btu Polystyrene has the next highest value at 5.00 ft2-hr-F/Btu

In wood door frame construction, the function of the shim is to (A) provide the required space for hardware (B) prevent sound from leaking through the opening (C) hide the gap between the frame and partition (D) provide for adjustibility in setting the frame plumb

(D) provide for adjustibility in setting the frame plumb A shim is a tapered piece of wood that, when used in pairs, allows the position of a door frame to be adjusted along the door's height until the frame is plumb

Which of the following design features would have the most detrimental effect on the environment of the surrounding neighborhood in an urban setting? (A) square building shape (B) dark exterior colors (C) extensive exterior paving (D) reflective glass

(D) reflective glass Reflective glass would subject the surrounding buildings, streets, and pedestrian areas to harsh reflective light and heat during most of the day. Moreover, it would affect the surrounding areas throughout the day and during all times of the year

Heat loss in a building can be minimized by selecting wall materials with high (A) conductance (B) enthalpy (C) permeability (D) resistance

(D) resistance Resistance is the number of hours needed for 1 Btu to pass through 1 ft2 of material of a given thickness when the temperature differential is 1 degree F. A higher resistance means that heat takes longer to pass through, and thus the material has a greater insulation value Conductance is the reciprocal of resistance and is the rate of heat loss measure in Btu/hr through 1 ft2 of material of a given thickness when the temperature differential is 1 degree F Enthalpy is the total heat in a substance, including latent heat and sensible heat Permeability is the property of a porous material that permits the passage of water vapor through it

An architect is designing custom oak cabinetry and wants the grain fo the door frames to be as straight and consistent as possible. Which type of sawing should be specified? (A) plain sawing (B) quarter sawing (C) flat sawing (D) rift sawing

(D) rift sawing

Which of the following would be most appropriate for damp-proofing on above-grade concrete wall with a moderately rough surface? (A) cementitious coating (B) bituminous coating (C) synthetic rubber (D) silicone coating

(D) silicone coating Silicone coatings would provide the best coverage for rough walls because they can be sprayed, painted, or rolled on If the wall was below grade, the correct choice would be a cementitious coating or a bituminous coating

With regard to the theories of proxemics, which approach to the design of a plaza would probably accommodate the most people while allowing individuals to maintain comfortable personal space distances? (A) large, flat, open space with trees defining the perimeter (B) heavily landscaped area with areas of lawn (C) amphitheater with bench-type seating in raised tiers (D) space with various levels, benches, and spaces defined by low walls

(D) space with various levels, benches, and spaces defined by low walls Proxemics deals with issues of territoriality, spacing and positioning between people, and how the organization of the environment can affect these issues A large, open space would encourage individuals and small groups to spread out rather than feel at ease nearer together A heavily landscaped area would be attractive and would help define separate spaces for people to gather, but may limit the greatest number of people from assembling An amphitheater would accommodate a large number of people but would be limited in flexibility for other types of informal uses

In regard to blast security, the space between a building and the outermost secured perimeter is called the (A) blast reduction zone (B) perimeter defense area (C) security setback (D) standoff distance

(D) standoff distance The standoff distance is the space between a building and the potential location of a blast threat. For blast protection, this distance should be maximized because blast energy decreases exponentially with increased distance between the source of the blast and the building

Retainage at substantial completion is frequently equivalent to (A) a percentage of incomplete items (B) the contractor's overhead (C) the cost of bonds (D) the contractor's profit

(D) the contractor's profit the amount of retainage at substantial completion is frequently comparable to the contractor's profit for the job - anywhere from 5-20%; commonly about 10%. Therefore, the contractor is often anxious to establish a substantial completion date and resolve any outstanding punch list items or other unresolved issues expeditiously, so that they may receive final payment

A deep excavation for a high-rise building in an urban area would require (A) battered walls (B) needle beams (C) steel sheeting (D) tiebacks

(D) tiebacks A deep excavation would require the use of vertical soldier beams supporting horizontal timber breast boards or cribbing. The vertical soldier beams must be anchored into the adjacent earth with grouted tieback rods. Even steel sheeting would require tiebacks for support A battered wall is simply a type of retaining wall using a material, such as stone or brick, slightly angled to support the adjacent earth. It is not appropriate for a deep excavation wall in an urban area where space is limited Needle beams are used to temporarily support a structure when its foundation is repaired or deepened Steel sheeting requires the use of rakers that extend into the excavation site, limiting the depth of the excavation and interfering with construction

A new restaurant has a maximum occupancy of 300 people. For all exits, the building code requires an allowance of 0.2 inches per occupant. Calculate the minimum number and size of exits (A) one exit, 5'-0" pair of doors (B) one exit, 6'-0" pair of doors (C) two exits, two 30" doors (D) two exits, two 3'-0" doors

(D) two exits, two 3'-0" doors 300 * 0.2 = 60 inches Because there are more than 50 occupants, two exits must be provided 60 inches / 2 = 30 inches Accessibility standards call for a minimum clear opening width of 32" for each door Hence, two exits, two 3'-0" doors

A building in a temperate climate will have some areas that require cooling and others that require heating at the same time. To minimize energy use, the bet devices to employ are (A) energy recovery ventilators (B) heat pipes (C) recuperative fuel exonomizers (D) water-loop heat pumps

(D) water-loop heat pumps Water-loop heat pumps use a continuous flow of temperate water to extract heat from areas that need to be cooled and add heat to other areas requiring heating using very little, if any, additional energy input Energy recovery ventilators work best in climates where the difference between indoor and outdoor air temperature is high Heat pipes are not appropriate for this use because they would simply pre-warm cool outdoor air A recuperative fuel economizer is another type of system that simply uses hot exhaust gas to preheat incoming air or water. This type of equipment would save energy by increasing the efficiency of the heating plant but would not be as effective as the water-loop heat pump system

The maximum allowable floor area of a building is primarily determined by the (A) city planning board (B) International Building Code (IBC) (C) Life Safety Code (D) zoning ordinance

(D) zoning ordinance The IBC limits the maximum allowable floor area of a building based on occupancy, building construction type, frontage, and whether the building is sprinklered, all of which may set a limit more or less than a zoning ordinance. A zoning ordinance sets limits on the floor area ratio based on lot size, setbacks, maximum height, and solar access, among other possible parameters. A combination of these factors will determine the maximum allowable floor area

A daycare facility is located in a temperate climate where wind can be problematic in the winter. Which trees should be planted to help mitigate the wind? (Choose the two that apply.) (A) Birch (B) Maple (C) Oak (D) Douglas fir (E) Pine

(D), (E) Coniferous trees, those with needle-like leaves, should be used to block wind and views. Of the listed items, douglas fir and pine are coniferous

A single-family residential building constructed entirely of unprotected wood framing would be which construction type? (A) Type I (B) Type II (C) Type III (D) Type IV (E) Type V

(E) Type V Type I is made entirely of noncombustible materials such as concrete or stone Type II is made of noncombustible materials with the exception of the roof covering material Type III has masonry-bearing walls but floors, structural framework, and roof made of wood or other combustible materials Type IV is heavy timber Type V is construction which is entirely combustible wood framed construction

How many different class types of cement are there?

5 Type I --- General purpose + General construction (most buildings, bridges, pavements, precast units, etc) Type II -- Moderate sulfate resistance + Structures exposed to soil or water containing sulfate ions Type III - High early strength + Rapid construction, cold weather concreting Type IV - Low heat of hydration (slow reacting) + Massive structures such as dams (now rare) Type V -- High sulfate resistance + Structures exposed to high levels of sulfate ions

Doors must swing in the direction of egress travel when the occupant load reaches what number?

50 Doors must swing in the direction of egress travel when the occupant load reaches 50. IBC §1008.1.2 Door Swing: "...Doors shall swing in the direction of egress travel where serving a room or area containing an occupant load of 50 or more persons or a Group H occupancy"

A one-story building with a total area of 10,000 ft2 is divided into a 7,000 ft2 restaurant (A2 occupancy) and a 3,000 ft2 office (B occupancy). The building is of type V-B construction and does not have a sprinkler system. The open space around the building gives it a 25% allowance for area increase. Does this building comply with IBC maximum area requirements?

No Check the maximum allowable area for both occupancy types using Aa == maximum allowable area At == allowable area factor NS == allowable area factor (non-sprinkler) If == area factor increase Sa == number of stories (1 in this case) Aa = (At + NS*If) (Sa) This gives us: Aa(A2) = 7,500 ft2 Aa(B) = 11,250 ft2 Now check the sum of the ratios of the actual building areas to the allowable building areas. This must be less than or equal to 1.0 sum = (7,000/7,500) + (3,000/11,250) sum = 1.20

Which of the following types of wood floors would be the least appropriate for a commercial office? (A) block (B) parquet (C) resilient (D) strip

Resilient would floors are commonly used for theater stages, dance floors, and gymnasiums. They provide extra bounce and resiliency for these types of uses

Find the coefficient of heat transmission for the wall assembly shown Exterior Air Film -- R=0.17 3 5/8" -- Brick -- R=.11/in 3/4" -- Air Space -- R=1.15 1/2" -- Insulating Sheathing -- R=4.3 3 1/2" -- Batt Insulation -- R=13 Air Barrier on warm side of insulation 1/2" -- Gypsum Board -- C=2.22 Interior Air Film -- R=.68

The R-values and C-values for the various components are given in the table. However, they must all be converted to conductance's for the thickness used. Air spaces and the thin layer of air on the exteriors and interiors of buildings have some thermal resistance. The brick has an R-value of 0.11 hr-ft2-F/Btu per inch and is 3 5/8" thick, so its total R-value is 0.40 hr-ft2-F/Btu Converting gypsum: R = = 1/C = 1/2.22 = 0.45 hr-ft2-F/Btu The sum of the R-values of the assembly then is Sum = 20.15 The overall coefficient of transmission, U, is: U = = 1/sum of R = 1/20.15 = 0.0496 Btu/hr-ft2-F


Set pelajaran terkait

Developmental Psychology (Life Span) Part 1

View Set

PrepU Chapter 36: Management of Patients with Musculoskeletal Disorders

View Set

Principles & Practices: Module 7 (Chapter 12, Overview Real Estate Finance)

View Set

Health Assessment: Rashid Ahmed Pre-Simulation Quiz

View Set

EC 402 Advanced Macroeconomics Final Exam

View Set

Mastering Biology Chapter 7 Study Module

View Set

Article 430 motors Article 440 air conditioning and refrigerating equipment

View Set

Final Exam International Marketing

View Set

Chapter 18 Neurologic Emergencies

View Set

Professional Selling Ch 1-9 Warm-Ups

View Set

Chapter 23: Management of Patients With Chest and Lower Respiratory Tract Disorders

View Set